El Concepto de Proporcionalidad.

Descargar como pdf o txt
Descargar como pdf o txt
Está en la página 1de 87

SECRETARÍ A DE EDUCACIÓN PÚBLICA

UNIVERSIDAD PEDAGÓGICA NACIONAL

UNIDAD UPN 095 AZCAPOTZALCO

El concepto de proporcionalidad, cómo lograr que los alumnos de segundo grado


de secundaria lo comprendan y lo apliquen

OFELIA VANESSA NÚÑEZ HERNÁNDEZ

DIRECTOR: DR. FRANCISCO JOSÉ ORTIZ CAMPOS

MÉXICO, D. F. 2014
SECRETARÍ A DE EDUCACIÓN PÚBLICA

UNIVERSIDAD PEDAGÓGICA NACIONAL

UNIDAD UPN 095 AZCAPOTZALCO

El concepto de proporcionalidad, cómo lograr que los alumnos de segundo grado


de secundaria lo comprendan y lo apliquen

Proyecto de intervención que para obtener el grado de

M A E S T R A E N E D U C A C I Ó N BÁ S I C A

PRESENTA:

OFELIA VANESSA NÚÑEZ HERNÁNDEZ

MÉXICO, D. F. 2014

2
3
A Dios:

Por darme la dicha de vivir día a


día, brindándome la suerte de
contar con personas excepcionales
en mi vida

Mami:

Eres la mejor de las mujeres,


gracias a ti, hoy puedo decir que
soy lo que soy, por todo aquello
que haz realizado por mí. Eres mi
ejemplo a seguir. Te amo

Mil gracias

A mis maestros:

Gracias por confiar en mí, por apoyarme


en el logro de esta meta, sin ustedes no lo
habría hecho.

De corazón gracias

4
INDICE

INTRODUCCIÓN .................................................................................................... 7

CAPÍTULO 1: CONTEXTO PROBLEMATIZADOR ................................................. 9

1.1 Política educativa ........................................................................................... 9


1.1.1 Unesco: cuatro pilares de la educación ................................................... 9
1.1.2 PISA y los resultados de matemáticas en los estudiantes de secundaria
en México ....................................................................................................... 10
1.1.3 La articulación del aprendizaje de las matemáticas y del concepto de
proporcionalidad ............................................................................................. 11
1.2 La reforma integral de la educación básica .................................................. 12
1.2.1 Reforma en educación preescolar 2004 ................................................ 13
1.2.2 Reforma en educación primaria 2009 .................................................... 14
1.2.3 Reforma en educación secundaria 2006 ............................................... 14
CAPÍTULO 2: FUNDAMENTACIÓN DE LA PROPUESTA DE INTERVENCIÓN . 20

2.1 Fundamentación teórica y didáctico – pedagógica de la propuesta de


intervención ........................................................................................................ 20
2.1.1 ABP como metodología de aprendizaje de las matemáticas ................. 20
2.1.2 El enfoque teórico de situaciones didácticas de Brousseau .................. 21
2.1.3 Caracterización ...................................................................................... 22
2.2 El concepto de proporcionalidad .................................................................. 24
2.3 Conocimiento cotidiano y conocimiento científico ........................................ 25
2.4 Planteamiento de la problemática ................................................................ 25
Capítulo 3: Propuesta de intervención .................................................................. 29

3.1 Elementos del diagnóstico ........................................................................... 29


3.2 Propósitos de la intervención ....................................................................... 32
3.3 Elementos metodológicos de la intervención ............................................... 32
3.3.1 Situación didáctica 1 .............................................................................. 34
3.3.2 Situación didáctica 2 .............................................................................. 34
3.3.3 Situación didáctica 3 .............................................................................. 34
3.3.4 Situación didáctica 4 .............................................................................. 35

5
3.3.5 Situación didáctica 5 .............................................................................. 35
3.3.6 Situación didáctica 6 .............................................................................. 35
3.3.7 Situación didáctica 7 .............................................................................. 36
3.4 Participantes ................................................................................................ 36
3.5 Descripción espacio-temporal ...................................................................... 39
3.6 Propuesta de intervención ........................................................................... 40
Capítulo 4.- Descripción y análisis de resultados .................................................. 51

4.1 Descripción de resultados ............................................................................ 51


4.1.1 Descripción de resultados por situación didáctica aplicada ................... 52
4.1.2 Análisis de resultados ............................................................................ 58
4.1.3 Análisis de la secuencia didáctica.......................................................... 74
Conclusiones ......................................................................................................... 76

Sugerencias .......................................................................................................... 78

Bibliografía ............................................................................................................ 79

ANEXO .................................................................................................................. 81

6
INTRODUCCIÓN

A partir de la experiencia con los 54 estudiantes del grupo de segundo grado de


secundaria, grupo dos, se observó que en 47 de ellos se presentaban dificultades
con la proporcionalidad ocasionando deficiencias en los aprendizajes de los
contenidos matemáticos y físicos, al no tener consistencia en los conceptos.

El interés por trabajar con el concepto de proporción se debe a que éste tiene
múltiples y variadas aplicaciones tanto en el campo de física como de la
matemática y desde luego en el ámbito de la vida cotidiana.

En el ámbito educativo se puede observar que existe dificultad para comprender el


concepto de proporción y para aplicarlo en la resolución de problemas. Cuando el
problema que se pretende resolver se refiere a una proporción que se puede
plantear de una manera directa es muy usual que los alumnos recurran a lo que se
conoce como regla de tres simple. Este procedimiento lo quieren hacer extensivo
a la resolución de problemas que dan lugar al planteamiento de una proporción
inversa, por lo que la aplicación del concepto de proporcionalidad se vuelve más
complejo ya que con la aplicación de regla de tres simple directa que conocen no
se resuelve el problema.

También se ha podido observar que en algunos casos se elaboran tablas de datos


que se obtienen por suma en cuyo caso no procede la aplicación de la tan
conocida regla de tres que los alumnos tienen mecanizada.

Originalmente se pensó aplicar la metodología que se conoce como Aprendizaje


Basado en Problemas (ABP) la cual consiste en el planteamiento de problemas
que implica la necesidad de investigar conceptos, procedimientos, trabajo en
equipo, socialización de lo investigado por cada uno de los participantes del
equipo, la discusión de los resultados obtenidos, la validación de dichos resultados
y la puesta de acuerdo para la presentación ante el grupo. Durante el desarrollo
del proyecto fue necesario transitar a una metodología que se conoce como

7
situaciones didácticas la cual consiste en el planteamiento de un problema para
que los alumnos lo resuelvan, en algunos casos con el apoyo del docente y en
otros sin contar con ese apoyo.

La ejecución de este proyecto de intervención se desarrolló durante el ciclo


escolar 2012 – 2013, con alumnos de segundo grado de educación secundaria.

Este proyecto consta de cuatro capítulos, en los cuales se muestra todo el proceso
que se llevó a cabo para aplicar la propuesta, en el primer capítulo titulado,
Contexto problematizador, se presenta todo el panorama tanto internacional como
nacional de la problemática investigada, se da cuenta de los orígenes del
contenido; mientras que en el segundo capítulo, se fundamenta la propuesta de
intervención, analiza el concepto de proporcionalidad y la justificación del trabajo
por situaciones didácticas.

En el tercer capítulo, Propuesta de intervención, se puede observar la secuencia


didáctica del contenido, el desarrollo de toda la aplicación, así como las
situaciones didácticas, aplicadas con el propósito de observar en los alumnos sus
procesos de conceptualización y aplicación de la proporcionalidad directa.

En el cuarto capítulo, Descripción y análisis de resultados, se procede a dar


cuenta de lo observado en todas y cada una de las situaciones didácticas más
representativas de la secuencia relacionada con el proceso de indagación del
proyecto. También se procede a la descripción, interpretación y valoración de los
resultados obtenidos.

Finalmente, se incluyen las conclusiones, recomendaciones y la bibliografía que


se utilizó como fundamento en el desarrollo de la argumentación teórica.

8
CAPÍTULO 1: CONTEXTO PROBLEMATIZADOR

1.1 Política educativa

1.1.1 Unesco: cuatro pilares de la educación


La educación es un derecho que todos tenemos, ya seas hombre, mujer, de
cualquier edad, color o religión. Contribuye a lograr un mundo más seguro, un
progreso personal y social; esto de acuerdo con la Conferencia Mundial sobre
Educación para Todos celebrada en marzo de 1990 en Jomtien, Tailandia, en la
cual, se reconoció que la educación que se imparte adolece de graves
deficiencias, por lo cual se requiere de adecuarla y ubicarla al alcance de todos,
convirtiéndose en un desafío y por lo cual se debe de poner énfasis en las
necesidades básicas del aprendizaje.

“Estas necesidades comprenden tanto las herramientas esenciales para el


aprendizaje (tales como lectura y escritura, aritmética, resolución de problemas)
como los contenidos básicos mismos del aprendizaje (conocimientos, aptitudes,
valores y actitudes) requeridos para que los seres humanos sean capaces de
sobrevivir; para que desarrollen sus capacidades intelectuales, vivan y trabajen
con dignidad, mejoren la calidad de sus vidas, tomen decisiones fundamentadas y
continúen aprendiendo” (UNESCO, 1994).

En la escuela tradicional la prioridad era que los estudiantes aprendieran


conocimientos factuales y conceptuales, de forma memorística y mecánica,
dejando de lado el desarrollo de otras habilidades, lo cual no ha ayudado en la
mejora del mundo. La educación es un instrumento fundamental en el desarrollo
de un país tanto intelectual como científico.

Al respecto es conveniente recuperar lo planteado por Delors:

“La educación debe estructurarse en torno a cuatro aprendizajes


fundamentales que en el transcurso de la vida serán para cada persona,
en cierto sentido, los pilares del conocimiento: aprender a conocer, es
decir, adquirir los instrumentos de la comprensión; aprender a hacer,
para poder influir sobre el propio entorno; aprender a vivir juntos, para
participar y cooperar con los demás en todas las actividades humanas;

9
por último, aprender a ser, un proceso fundamental que recoge
elementos de los tres anteriores” (Delors, 1996).
Los docentes deben de orientar la educación al desarrollo integral de las
personas, aprender a aprender en donde el estudiante desarrolla su creatividad,
despertando así su curiosidad intelectual mediante el análisis de la realidad,
fomentado su autonomía de juicio, que implica un aprender a hacer para impulsar
su iniciativa al trabajo de manera cooperativa para el desarrollo de competencias
específicas, una vez adquiridos estos aprendizajes el estudiante debe aprender a
vivir en convivencia, propiciando su identidad ciudadana para aprender a ser una
persona con libertad responsable y autoconsciente.

En el Foro Mundial sobre la Educación celebrado en Dakar, Senegal en el año


2000 (UNESCO, 2000), se ha insistido en una educación que oriente a explotar los
talentos y las capacidades de cada persona y desarrollar la personalidad del
educando, con objeto de que mejore su vida y transforme la sociedad. Con lo
anterior, se puede afirmar que la finalidad de la educación es la formación integral
del ser humano.

1.1.2 PISA y los resultados de matemáticas en los estudiantes de secundaria


en México
En nuestro sistema educativo, existe una influencia política y económica en el
marco de la educación por organismos internacionales como la Organización para
la Cooperación y el Desarrollo Económico (OCDE), el Banco Mundial y el Fondo
Monetario Internacional, quienes aportan el presupuesto a cambio de estar
presentes en nuestro país, dando resultados en el ámbito educativo, que permiten
comparar los resultados educativos de diferentes países, tal es el caso del
proyecto DeSeCo (Definición y Selección de Competencias) que tiene como
objetivo definir y seleccionar las competencias consideradas esenciales para la
vida de las personas y el buen funcionamiento de la sociedad.

Un instrumento de evaluación es la prueba estandarizada PISA, por sus siglas en


inglés, (Programa para la Evaluación Internacional de los Alumnos) que se lleva a
cabo cada tres años desde 1997, en nuestro país, tiene como finalidad reflejar el

10
nivel de avance en cuanto a las competencias que los estudiantes de 15 años de
edad han adquirido en su trayecto formativo en las asignaturas de Español,
Matemáticas y Ciencias.

Los resultados obtenidos en dichas pruebas estandarizadas como son PISA y


ENLACE, han sido también causas de las diferentes modificaciones en el sistema
educativo.

Durante el periodo de gobierno 2006 - 2012, se realizó un convenio con el


magisterio, llamado Alianza por la Calidad de la Educación, en él se postula la
articulación de la educación básica, que en ese periodo se decretó en el acuerdo
592, con un enfoque por competencias, centrándose en los procesos de
aprendizaje de las alumnas y los alumnos, al atender sus necesidades específicas
para que mejoren las competencias que permitan su desarrollo personal.

Sin embargo, una característica que ha permeado en las distintas modificaciones


realizadas al currículo o plan de estudios es que no existe un seguimiento bien
definido, debido a que no se da el tiempo necesario para analizar los resultados
que este arroja, al no tener una manifestación inmediata, y no responder a una
necesidad pedagógica, sino a la dinámica que la política educativa asume cada
ciclo presidencial. (Barriga, 2006)

1.1.3 La articulación del aprendizaje de las matemáticas y del concepto de


proporcionalidad

En la cotidianeidad las matemáticas son tan prácticas que sin darte cuenta las
realizas en todo momento desde que despiertas y se observa el reloj , cuando ves
las raciones de comida que emplearás para al almuerzo, hasta cuando vas de
paseo, sin embargo todos estos conocimientos se formalizan en la escuela. Y es
aquí donde los niños, adolescentes y jóvenes inician con el prejuicio de la
enseñanza de las matemáticas.

11
La proporcionalidad es un contenido importante en la asignatura de Matemáticas,
debido a que se encuentra en distintos contextos, de acuerdo con Ramírez y
Block, 2009, citado en Mochón Cohen (Mochón Cohen, 2012) la mayoría de las
actividades matemáticas de nuestra vida cotidiana están basadas en este
concepto por ser el más sencillo de utilizar (5 piezas cuestan 5 veces lo que una
pieza). Sin embargo, las ideas de proporcionalidad son en general mal entendidas,
debido a que es común que en el aula se enseñe este tema de manera mecánica
utilizando la regla de tres.

Este contenido se observa formalmente en el cuarto año de educación primaria y


continua su estudio hasta la educación secundaria, éste se ubica en el eje
temático Manejo de la información debido a que los estudiantes deben partir de la
información que tienen para comprender y analizar cuál es el procedimiento más
adecuado para dar solución a la situación planteada.

1.2 La reforma integral de la educación básica

En la búsqueda de caminos que conduzcan al éxito y trabajar para la mejora de la


educación, se implementa La Reforma Integral de la Educación Básica (RIEB),
que tiene como base al artículo 3º de nuestra Carta Magna, del Plan Nacional de
Desarrollo 2007-2012, justificado en el eje 3. Igualdad de oportunidades, donde
nos marca el punto 3.3 la transformación educativa, mientras que el Programa
Sectorial de Educación en su objetivo 4 busca ofrecer una educación integral en el
nivel básico, además de considerar el Acuerdo por la Calidad de la Educación con
el que se propone impulsar una transformación hacia la calidad

El plan de estudios 2011, está constituido por la articulación de la educación


básica, es decir que debe existir una continuidad entre Preescolar, Primaria y
Secundaria, con el fin de que al concluir la educación básica los estudiantes
obtengan el perfil de egreso deseado como antecedente para el nivel medio
superior.

12
La articulación en la educación significa lograr la unidad de ideas y acciones, en
donde todos los profesores deben de estar involucrados, porque es preciso
coordinar el trabajo y las actividades a fin de unificar criterios y modos de
actuación; dicha articulación debe proporcionarnos un desarrollo progresivo e
integral en los alumnos.

Dentro de los propósitos que la RIEB indica esta el ofrecer a los estudiantes un
trayecto formativo coherente y de profundidad de acuerdo con sus niveles de
desarrollo, sus necesidades educativas y expectativas para el futuro además de
transformar y mejorar las prácticas pedagógicas, orientándolas a favorecer en los
niños el desarrollo de competencias.

1.2.1 Reforma en educación preescolar 2004

Los cambios en Educación Preescolar en 2004, comienzan a introducir el


concepto de competencias en la educación básica, seguido dos años después con
la Educación Secundaria y culminando en 2009 con la Educación Primaria. Los
cambios anteriores se hicieron con la finalidad de transformar las prácticas
educativas, para un mejor desempeño de los estudiantes y la adquisición de
competencias.

Pero ¿qué es competencia? Para DeSeCo (Zabala & Arnau, 2007) es la


capacidad de responder a demandas complejas y llevar a cabo tareas diversas de
forma adecuada. Supone una combinación de habilidades prácticas,
conocimientos, motivación, valores éticos, actitudes, emociones y otros
componentes sociales y de comportamiento que se movilizan conjuntamente para
lograr una acción eficaz.

Mientras que para Argudín (Argudín, 2009), la competencia es un: “conjunto de


comportamientos sociales, afectivos y habilidades cognoscitivas, psicológicas,
sensoriales y motoras que permiten llevar a cabo adecuadamente un papel, un
desempeño una actividad o una tarea”

13
De esta forma la educación gira en torno al enfoque por competencias, en donde
el estudiante es capaz de movilizar sus saberes conceptuales, procedimentales y
actitudinales en su entorno social.

Competencia: es la capacidad de responder a diferentes situaciones, e implica un


saber hacer (habilidades) con saber (conocimiento), así como la valoración de las
consecuencias de ese hacer (valores y actitudes) (SEP, Plan de estudios.
Educación Básica, 2011)

1.2.2 Reforma en educación primaria 2009

Mientras que en la educación primaria dicha reforma se realizó cinco años más
tarde, a la educación preescolar y tres años después de secundaria, de ahí surge
una interrogante, por qué no seguir el orden jerárquico y de esta manera la
primaria no debe ser la última en reformarse sino en segundo lugar, además de
que se puede observar el avance de los estudiantes.

Otro de los inconvenientes observados es que la reforma fue aplicada por


fragmentos, al iniciar con los alumnos de primer y sexto año, seguido de segundo
y quinto grado, culminando los cambios en tercero y cuarto, por lo cual en un inicio
las expectativas en el nivel secundaria no fueron las esperadas, debido a que un
solo año trabajaron con los nuevos programas, sin embargo poco a poco se fue
observando el cambio.

1.2.3 Reforma en educación secundaria 2006

Mientras que en secundaria la reforma fue llevada a cabo en el año 2006,


existieron varios cambios entre ellos la eliminación del término asignaturas
cocurriculares, entre las cuales se encontraban Educación Física, Educación
Artística entre otras, en estos momentos todas las materias tienen el mismo valor.

14
Los contenidos que pertenecen a la ciencia (Biología, Física y Química), se unen
en una sola asignatura que es Ciencias y cada grado tiene un énfasis específico,
además se aumenta el número de horas a la semana siendo ahora de seis horas.

Otra de las modificaciones, es el cambio en la organización de los contenidos, por


ejemplo, Español se divide en ámbitos y la secuencia se realiza por medio de
proyectos en los cuales se abarcan varios temas y no uno sólo como
anteriormente, en donde se veía tema por tema. Lo que se busca es relacionar
varios contenidos para que no sean tan repetitivos y los cuales tienen que
relacionarse con el proyecto planeado.

1.2.3.1Campos formativos

A la organización del mapa curricular se agregan nuevas directrices llamadas


campos formativos (Plan de Estudios 2011)

• Lenguaje y comunicación. Desarrolla competencias comunicativas y de


lectura.
• Pensamiento matemático. Desarrolla el razonamiento para la solución
de problemas.
• Exploración y comprensión del mundo natural y social. Integra diversos
enfoques disciplinares, constituye la base de la formación del
pensamiento.
• Desarrollo personal y para la convivencia. Integra diversos enfoques
disciplinares para un desarrollo más pleno e integral de las personas.

15
Figura 1

Figura 1. Mapa curricular de la educación básica. Fuente: SEP

Los Programas de estudio 2011 se integran por propósitos, enfoques, Estándares


Curriculares y aprendizajes esperados, manteniendo su pertinencia, gradualidad y
coherencia de su contenido, y cada tres años serán evaluadas de acuerdo a los
periodos en su cartilla de educación básica, comenzando al término de la
educación Preescolar, tercero de Primaria, sexto de primaria y tercer grado de
secundaria.

Con base en lo anterior, el rol del docente pasó a ser solo el mediador, el nexo
que hay entre el conocimiento y el alumno. Desde esta postura piagetiana, se
considera que el sujeto produce su conocimiento como resultado de la interacción
con el medio, y el docente se hace mediador, en tanto permita el desarrollo de la
actividad matemática del alumno.

1.2.3.2 Pensamiento Matemático


El estudio de las matemáticas se ve implicado durante todo el proceso educativo,
desde la educación preescolar hasta la culminación de sus estudios, mismos que
son fundamentales para todo el ser humano debido a que se desarrollan

16
competencias, destrezas y un nivel de conocimiento necesario para poder dar
respuesta a las condiciones del entorno social.

La disciplina de las Matemáticas, es una herramienta fundamental en el desarrollo


de estudios científicos y técnicos, por tal motivo, es que la raza humana tiene la
necesidad constante de crear y fortalecer sus conocimientos matemáticos, no sólo
para los profesionales o los especialistas en diversas materias, sino para toda la
comunidad, debido a que se emplean cotidianamente, desde que vas a la tienda y
compras hasta la construcción de una casa (SEP, Libro para el maestro.
Matemáticas. Secundaria, 1994).

1.2.3.3 Articulación vertical de contenidos


La asignatura de Matemáticas se comienza a trabajar desde la Educación
Preescolar, ubicándose en el campo formativo Pensamiento Matemático, en
donde a los niños se les presentan los números, la cantidad que representa cada
uno de ellos, las figuras básicas como son el cuadrado, círculo, rectángulo y
triángulo, estudiando de ellos la forma y el espacio que ocupan, comenzando así
la relación estrecha con las matemáticas.

Hay continuidad entre la Educación preescolar y la Primaria, es en esta última,


donde el estudio de las matemáticas toma otro sentido, dando mayor énfasis al
proceso de solución en la resolución de problemas, el alumno desarrolla la
capacidad de utilizarla como un instrumento para reconocer, plantear y resolver
problemas siendo esto producto del quehacer humano y del proceso de
construcción en las abstracciones y experiencias de los niños.

En la educación secundaria, y con el fin de una mejora educativa en nuestro país,


los planes y programas de estudio vigentes, se enfocan principalmente en el
desarrollo de competencias y habilidades en el alumno. En este sentido la
Reforma a Secundaria pretende “responder a los requerimientos formativos de los
jóvenes de las escuelas secundarias, para dotarlos de conocimientos y
habilidades que les permitirán desenvolverse y participar activamente en la
construcción de una sociedad”. Esta reforma de acuerdo con el Programa de
Matemáticas (SEP, Programa de estudios. Matemáticas, 2006) tiene como

17
planteamiento central “llevar a las aulas actividades de estudio que permitan
encontrar diferentes formas de resolver los problemas y formular argumentos que
validen los resultados. El conocimiento de reglas, algoritmos, fórmulas y
definiciones sólo es importante en la medida en que los alumnos lo puedan usar
de manera flexible para solucionar problemas”, es decir que sea significativo en su
vida cotidiana. En el plan y programas se consideran tres elementos
fundamentales para el logro del perfil de egreso:

• Los alumnos: el punto central de atención, conociendo así los procesos


cognitivos de los sujetos que aprenden.

• El profesor: es un actor primordial, pues es un mediador entre el


conocimiento y el alumno, haciendo uso de la didáctica definida como el arte de
enseñar.

• El conocimiento matemático: se sabe que el proceso enseñanza-


aprendizaje es el principal objeto de estudio de esta concepción clásica de la
didáctica; de ahí se derivan enfoques posteriores como el de “Aprendizaje
significativo”, cuyo objeto de estudio es el conocimiento matemático del alumno y
su evolución, aunque la explicación de este proceso se delega a la psicología

La construcción del conocimiento por parte del alumno debe ser un proceso de
estudio que se va a ir formando a partir de lo que el alumno conoce y las
reflexiones que éste vaya teniendo, considerando que las situaciones
problemáticas deben hacer uso de las herramientas matemáticas que se pretende
estudiar, así como los procesos que siguen los alumnos para construir nuevos
conocimientos y superar las dificultades que surgen en el proceso de aprendizaje.

Las competencias matemáticas de acuerdo al Programa de Matemáticas 2011


(SEP, Plan de estudios. Educación Básica, 2011):

• Resolver problemas de manera autónoma

• Comunicar información matemática

18
• Validar procedimientos y resultados

• Manejar técnicas eficientemente.

Se sigue dividendo el programa en tres ejes temáticos: Sentido Numérico y


Pensamiento Algebraico (SN y PA), Forma, Espacio y Medida (FEM), Manejo de
la Información (MI), con nueve temas en total y para cada uno de ellos, un
estándar curricular que los alumnos deben cubrir al finalizar cada tema.

Las matemáticas no sólo sirven para resolver problemas aritméticos, algebraicos o


geométricos sino también para entender mejor los fenómenos naturales además
de los artísticos.

19
CAPÍTULO 2: FUNDAMENTACIÓN DE LA PROPUESTA DE INTERVENCIÓN

2.1 Fundamentación teórica y didáctico – pedagógica de la propuesta de


intervención

Tema:

El concepto de proporcionalidad, cómo lograr que los alumnos de segundo grado


de secundaria lo comprendan y lo apliquen

2.1.1 ABP como metodología de aprendizaje de las matemáticas

Para llevar a cabo esta intervención en un inicio se tenía planeado trabajar con
esta metodología del Aprendizaje basado en problemas (ABP) pero en la
intervención se optó por trabajar con el enfoque teórico de situaciones didácticas
de Brousseau.

ABP es una estrategia de enseñanza – aprendizaje para la adquisición de


conocimientos, el desarrollo de habilidades y actitudes a través de la resolución de
problemas. (Guevara Mora, 2010).

Es un método de aprendizaje basado en el principio de utilizar problemas como


punto de partida para la adquisición e integración de nuevos conocimientos.

Los principios básicos del ABP son:

• Entendimiento al ser una situación de la vida real


• Conflicto cognitivo estimula el aprendizaje
• Conocimiento se desarrolla con el reconocimiento y aceptación

Sus objetivos son:

• Promover en el alumno la responsabilidad de su aprendizaje


• Desarrollar habilidades para la evaluación critica
• Desarrollar habilidades para las relaciones interpersonales

20
• Involucrar al alumno en un reto
• Desarrollar el razonamiento
• Orientar la falta de conocimiento y habilidades de manera eficiente y
eficaz
• Estimular el desarrollo del sentido de colaboración

Para llevar a cabo la estrategia los estudiantes se reúnen en equipos no mayores


a cuatro estudiantes, el docente solo es el mediador, además de observar el
avance en el desarrollo del aprendizaje de los estudiantes. Se comienza por
plantear un problema, el cual tiene que ser motivador e interesante más aun si se
trata de una situación de la vida real, ya que esto les contribuirá en la toma de
decisiones, realizar juicios para argumentar la solución al problema, e involucra a
todos los integrantes, pero ¿Qué deben hacer los alumnos al enfrentarse a un
problema?

2.1.2 El enfoque teórico de situaciones didácticas de Brousseau

“Para resolver la situación, el alumno debe usar sus conocimientos previos,


mismos que le permiten entrar en la situación, pero el desafío consiste en
reestructurar algo que ya sabe, sea para modificarlo, ampliarlo, rechazarlo o para
volver a aplicarlo en una nueva situación” (SEP, Plan de estudios. Educación
Básica, 2011)

La situación didáctica es una situación construida intencionalmente con el fin de


hacer adquirir a los alumnos un saber determinado. Brousseau, en 1982, la definía
de esta manera (Gálvez, 1994).

“Un conjunto de relaciones establecidas explícita y/o explícitamente


entre un alumno o un grupo de alumnos, un cierto medio (que
comprende eventualmente instrumentos u objetos) y un sistema
educativo (representado por el profesor) con la finalidad de lograr que

21
estos alumnos se apropien de un saber constituido o en vías de
constitución.”

De acuerdo al enfoque por competencias, para lograr que estas se desarrollen, se


debe crear una situación concreta y definida, ante esto, surge la interrogante ¿qué
es un situación? para Roegiers (Roegiers & Peyser, 2007) evoca un conjunto
contextualizado de informaciones que un alumno deberá articular a fin de resolver
una tarea determinada, presentando un obstáculo, un desafío cuya solución
permitirá nuevos aprendizajes.

Con base en Brousseau (Brousseau, 2007) en su teoría de las situaciones


didácticas se menciona que una situación es un entorno del alumno, diseñado y
manipulado por el docente, que sirve como herramienta, para desarrollar sus
habilidades en el proceso de solución.

Según Brousseau (Sadovsky, 2005) “El alumno aprende adaptándose a un medio


que es factor de contradicciones, de dificultades, de desequilibrios, un poco como
lo ha hecho la sociedad humana. Este saber fruto de la adaptación del alumno, se
manifiesta a través de respuestas nuevas que son la prueba del aprendizaje”

2.1.3 Caracterización

2.1.3.1 Características
Santos (Santos Trigo, 1997), sugiere que las situaciones propuestas a los
alumnos incluyan un reto, tengan un propósito determinado además de que el
esquema de razonamiento que contenga dicha situación sea diferente, para que el
alumno se interese por dicha situación de lo contrario pasara desapercibida y no
lograra el objetivo planteado.

Lo importante de una situación didáctica no es solamente el problema a resolver,


sino la interacción entre el alumno y el conocimiento, para lo cual se tienen cuatro
momentos:

• Las situaciones de acción, en donde se les presenta a los estudiantes el


conflicto cognitivo y comienzan a tomar decisiones.

22
• Las situaciones de formulación, el alumno conoce la problemática y
comunica la información a sus compañeros

• Las situaciones de validación, comprueba que su respuesta es la correcta.

• La institucionalización es la acción por la cual el profesor atribuye a un


conocimiento aprendido mediante las situaciones escogidas la condición de objeto
matemático digno de interés científico.

2.1.3.2 Roles (profesor – alumno)


La devolución es la acción mediante la cual el profesor traspasa al alumno la
responsabilidad de la situación que le propone con relación a un determinado
conocimiento, aceptando aquél la responsabilidad de esa transferencia. Esta
acción tiene lugar en el seno de la negociación de un contrato muy particular: el
contrato didáctico.

Se sabe que el juego también es una estrategia de aprendizaje y quizá la más


efectiva, debido a que los estudiantes les agrada más jugar, palpar los objetos y le
es más significativo de ahí que Brousseau introdujo el término de situación a-
didáctica

“El término de situación a-didáctica designa toda situación que, por una parte no
puede ser dominada de manera conveniente sin la puesta en práctica de los
conocimientos o del saber que se pretende y que, por la otra, sanciona las
decisiones que toma el alumno (buenas o malas) sin intervención del maestro en
lo concerniente al saber que se pone en juego.” (Brousseau, 2007)

El contrato didáctico se refiere a la negociación establecida entre profesor y


alumno. Comprende el conjunto de comportamientos que el profesor espera del
alumno y los que el alumno espera del docente

23
2.2 El concepto de proporcionalidad

Otro contenido importante para el proyecto es el concepto de proporcionalidad, de


ahí que se requiera conocer sus raíces.

Según Freudenthal (1978) (Puing, L. 1997)., en su fenomenología didáctica,


menciona que la fracción es un recurso del número racional, estos números son
de orden superior a los naturales y enteros, siendo los naturales los números más
simples, seguidos de los enteros al incluirse los números negativos y el cero,
adicionalmente se incluyen los números irracionales, que junto con los racionales
dan lugar al conjunto de los números decimales que conocemos como números
reales.

La fracción puede tener como función también el de fracturar, cuando se toman


ciertas partes del entero o total, operador, cuando se lleva a cabo una operación
con los racionales y comparador, al orientar su enfoque en las similitudes y
diferencias entre las cantidades. La razón es una función de un par ordenado de
números o valores de magnitud, también se puede considerar como la
comparación por cociente de dos cantidades medibles, por tal motivo puede ser
representada con una fracción, empero no debemos olvidar que sólo es una forma
de escribirse, no una fracción y se leen de manera distinta.

𝑎
𝑎: 𝑏 se lee: a es a b
𝑏

A la relación de igualdad entre dos razones se le denomina proporción.


𝑎 𝑐
∷ 𝑎: 𝑏 ∷ 𝑐: 𝑑 se lee: a es a b como c es a d
𝑏 𝑑

De acuerdo a Piaget (1974) (Puing, L. 1997)., la noción de proporcionalidad se


encuentra en el nivel de las operaciones formales, es decir, se trata de
operaciones sobre operaciones. Va más allá de una operación básica como la

24
suma, la resta, la multiplicación o la división, al requerir de una comprensión más
fina de equivalencias con números más complejos, al ser infinitos.

Analizar el Identificar la Elaborar un


Identificar los información
escenario del con la que se esquema del
objetivos
problema cuenta problema

Diseñar un Realizar un
Analizar la Recopilar
esquema de diagnostico
información información
trabajo situacional

Plantearse Retroalimentar
La evolución
los resultados del grupo

Figura 2. Las actitudes de los alumnos frente a los problemas

2.3 Conocimiento cotidiano y conocimiento científico

De acuerdo con la experiencia obtenida en la labor docente se puede definir al


conocimiento cotidiano como aquello nato, que se adquiere con el paso de los
años y con ayuda de la observación, pues con todos los acontecimientos vividos
por cada ser humano, éste va aprendiendo de aquello que experimenta; mientras
que el conocimiento cotidiano es la formalización e institucionalización de lo que
cada uno conoce, pero que hasta este momento adquiere un nombre nuevo,
además de que sus estructuras se fortalecen en cuestión del aprendizaje.

2.4 Planteamiento de la problemática

La proporcionalidad es un contenido de aplicación multidisciplinar al yuxtaponer


disciplinas diversas sin relación aparente, pero que constituyen una enseñanza, tal
es el caso de Física, Química y Biología. Además de ser un contenido matemático
que tiene distintas aplicaciones en los diversos contextos en donde nos

25
desenvolvemos, al estar implícitas operaciones como la multiplicación y la división
que dan lugar a fracciones, decimales, etc.

La proporción es entendida como la relación entre dos conjuntos de cantidades o


razones, es proporcional directa si existe un número al que se le llama constante
de proporcionalidad, siempre el mismo que multiplicando a cualquiera de las
cantidades de un conjunto da como resultado la cantidad correspondiente del otro
conjunto.

“El planteamiento central en cuanto a la metodología didáctica que se sugiere para


el estudio de las Matemáticas, consiste en utilizar secuencias de situaciones
problemáticas que despierten el interés de los alumnos y los inviten a reflexionar,
a encontrar diferentes formas de resolver los problemas y a formular argumentos
que validen los resultados. Al mismo tiempo, las situaciones planteadas deberán
implicar justamente los conocimientos y las habilidades que se quieren desarrollar”
(SEP, Plan de estudios. Educación Básica, 2011)

El docente debe de realizar una secuencia en tres momentos, primero diseñando


el problema, estableciendo las reglas del trabajo, e identificar los momentos de
intervención, después identificar los puntos clave, formular hipótesis por ultimo
establecer los planes del aprendizaje.

Piaget: aprendemos cuando enfrentamos un conflicto cognitivo que modifique


nuestros esquemas.

A partir de lo anterior surge la inquietud de trabajar en la comprensión de la


proporcionalidad no sólo adición siendo también regla de tres. Al ser un
instrumento que fundamenta el trabajo de investigación de la ciencia, el
establecimiento y comprobación de leyes, solución de problemas, es la relación de
proporcionalidad entre las magnitudes intervinientes. Por lo cual se espera
erradicar las dificultades que los estudiantes presentan en su aprendizaje, esto se
llevará a cabo a través de la intervención docente que mi persona realice con el fin
de modificar a bien el concepto de proporcionalidad y la comprensión del mismo.

26
Lo que es más sorprendente es que a pesar de ser un tema bastante visto, desde
primaria, y que en secundaria en el primer año se ve valor unitario, constante de
proporcionalidad o regla de tres sigan cometiendo los mismos errores, y cuando
se enfrentan ante un problema que implica la proporcionalidad, los estudiantes
sólo sumen o resten la diferencia que existe entre los datos.

Se espera lograr realizar una secuencia didáctica en la que se observe


detalladamente lo que es la proporcionalidad, recordando que ésta puede ser
directa, indirecta o múltiple

El plantear un problema no es una tarea fácil para algunos alumnos, el trabajar


con situaciones resulta una actividad complicada y más aún cuando se le solicita
que las interprete o comprenda, generando en ellos confusión en el proceso de
solución.

El propósito que se plantea en el presente proyecto de intervención es: partir de


una secuencia didáctica en donde el alumno comprenda y aplique el
contenido de proporcionalidad a otras áreas ya sea de su vida cotidiana o
del conocimiento

De tal forma, que para lograr llegar a dicho propósito, fue necesario identificar los
distintos errores en los que más incurren los estudiantes al resolver o plantear un
problema, dando pauta al tratamiento del análisis y reflexión de los resultados
obtenidos, para conocer el avance en el proceso de aprendizaje, además de la
búsqueda de otras estrategias que puedan apoyar en la adquisición de
conocimientos.

Para poder llevar a cabo el propósito planteado, se diseñará un conjunto de


situaciones didácticas que permita contestar las siguientes preguntas que guiarán
el presente trabajo:

 ¿cuál es el concepto que los alumnos tienen de proporcionalidad?


 ¿cómo relacionan el concepto de razón con proporción?

27
 ¿La regla de tres es la mejor técnica para resolver problemas de
proporcionalidad?
 ¿si el alumno comprende el concepto de proporcionalidad, logrará aplicarlo
en problemas cotidianos?

28
Capítulo 3: Propuesta de intervención

3.1 Elementos del diagnóstico


Son muchos los estudiantes que se resisten al aprendizaje de las matemáticas, tal
vez por el miedo a errar, por la dificultad en su lenguaje o por los comentarios
externos de sus compañeros. Para evitar dichas causas, el docente debe de
fomentar ambientes de aprendizaje óptimos para la adquisición del conocimiento,
favoreciendo la comunicación y la confianza entre los alumnos de manera que
puedan expresar su creatividad.

Siempre es necesario conocer el nivel en el cual los estudiantes llegan a un nuevo


ciclo escolar, para identificar si cuentan con los antecedentes necesarios que
permitan avanzar en la construcción del conocimiento y trabajar en el desarrollo de
sus competencias, además de conocer características personales de los
estudiantes.

Se llevó a cabo el diseño de un examen escrito, con el propósito de conocer


cuáles eran las dificultades más relevantes en el aprendizaje de la asignatura de
Matemáticas, asimismo indagar sobre las estrategias que puedan evitar dicha
problemática. El instrumento se diseñó con 8 reactivos, de los cuales cinco eran
de opción múltiple y tres situaciones en las cuales debían de dar una respuesta
mostrando el procedimiento para obtenerla. Los contenidos que se evaluaron
fueron conversión de unidades, sustitución de datos, proporcionalidad, volumen,
trazos de rectas y ángulos (Anexo 5). Este instrumento se aplicó a tres grupos,
dos de matemáticas y uno de ciencias, encontrándose resultados muy
semejantes. El problema de la proporcionalidad fue el que dio lugar a un mayor
número de respuestas diferentes al resultado esperado.

El problema de proporcionalidad planteado fue el siguiente:

El peso de los objetos varía en función de la fuerza de gravedad que actúa


sobre ese objeto. Si un objeto que en la tierra pesa 4 kg, en júpiter pesa 10 kg.
¿Cuánto pesará en júpiter un objeto que en la tierra pesa 12 kg?

a) 6 kg b) 18 kg c) 30 kg d) 4.8 kg

29
Los resultados obtenidos se muestran en la siguiente tabla:

Tabla 1. Resultados obtenidos en el examen de diagnóstico, problema dos,


aplicado a tres grupos

El análisis de las respuestas que dieron en relación con cada una de las opciones
da lugar al siguiente análisis.

A. La respuesta es incorrecta, el alumno sólo se percató que existe una


diferencia de 6 kg entre la Tierra y Júpiter, pensando así que para obtener
el resultado bastaba con restar 6 al valor que se tiene en Júpiter para
obtener la respuesta. En este caso no existió una buena lectura, porque los
12 kilogramos son en la Tierra no en Júpiter.
B. La respuesta es incorrecta, en este inciso el alumno se pudo percatar de
que existe una diferencia de 6, pues si a cuatro le suma 6 obtiene como
resultado 10, así que, sí a 12 le suma 6 obtiene 18 como resultado, pero no
verifico que debe existir una proporción.
C. La respuesta es correcta

Para obtener esta respuesta pudo recurrir a lo que se conoce comúnmente


como regla de tres.
D. En esta respuesta, el alumno realizó una regla de tres, pero no realizó las
operaciones bien porque no colocó los datos en el orden correcto de
acuerdo a las unidades de medida, de manera que en lugar de multiplicar
10 x 12, él multiplicó 12 x 4 y posteriormente lo dividió entre 10.

30
En la siguiente gráfica se puede observar que sólo uno de cada siete alumnos
(13%) obtuvo la respuesta correcta que corresponde al inciso C.

5%
a) 6 kg
13% 18%
b) 18 kg

c) 30 kg
64%
d) 4.8 kg

Gráfica 1. Resultados obtenidos del problema 2.

De acuerdo con lo anterior, me pude dar cuenta que los alumnos mostraron
mayores dificultades para resolver el contenido de proporcionalidad. Ellos no
tenían claro que el proceso de resolución implicaba más de una operación básica.
Lo anterior me llevó a analizar y razonar los resultados obtenidos con el propósito
de lograr comprender cuál fue el procedimiento que emplearon para obtener la
respuesta. En la mayoría de los casos me pude dar cuenta de que los alumnos no
lograban entender que el enunciado se refería al establecimiento de una relación
constante.

De este modo, el presente proyecto aborda el concepto de proporcionalidad, con


el fin de que los alumnos lo comprendan y lo miren con otra perspectiva, que les
permita aplicarlo para dar respuesta a las múltiples situaciones que se le
presenten en su vida diaria.

Antes de iniciar con la aplicación del instrumento de diagnóstico, se preguntó a los


estudiantes si la asignatura era de su agrado o no, se encontró que un 65%
opinaban que no les gustaba la Matemática. Algunas de las razones de esa

31
opinión, fueron las siguientes: que son muy difíciles, que siempre ha sido su coco,
que no les entienden, etc.

3.2 Propósitos de la intervención

El contenido matemático (proporcionalidad), es muy complejo, al ser más que una


simple operación básica, implica un razonamiento para poder efectuarlo y obtener
por tanto su resultado, además de estar presente en nuestra vida cotidiana, al
realizar una comida (receta de cocina), al obtener una ampliación o una reducción
de un documento, al ir de compras, en las rebajas, en las leyes de la física, entre
otras.

Por lo anterior se ha decidido trabajar sobre la comprensión de la proporcionalidad


directa, teniendo como propósitos:

 Partir de una situación didáctica en donde el alumno comprenda y


comunique el contenido de proporcionalidad a otras áreas ya sea de su vida
cotidiana o del conocimiento.
 Comprender el concepto de proporcionalidad para aplicarlo en los distintos
contextos que se le presenten.
 Favorecer en el alumno el gusto por la materia.

3.3 Elementos metodológicos de la intervención


De tal forma, que para alcanzar los propósitos planteados, fue necesario identificar
los distintos errores en los que más incurren los estudiantes al resolver o plantear
un problema, dando pauta al tratamiento del análisis y reflexión de los resultados
obtenidos, para conocer el avance en el proceso de aprendizaje, además de la
búsqueda de otras estrategias que puedan apoyar en la adquisición de
conocimiento.

32
Para tratar de lograr los propósitos anteriores se diseñó una secuencia didáctica
con seis situaciones didácticas aplicadas que corresponden al planteamiento de
problemas cuyos aprendizajes esperados son: Resuelve problemas vinculados a
la proporcionalidad directa (porcentajes, escalas).

Las situaciones didácticas planteadas como problemas son las siguientes:

33
3.3.1 Situación didáctica 1
La cuadrícula

En cierta ocasión Carolina fue de compras a una tienda de artesanías ubicada en


el centro de Toluca, ahí encontró unos cuadros muy bonitos, los observó
detenidamente comprendiendo que los habían hecho sobre una cuadrícula, he
aquí los cuadros.

1. ¿Cuántos cuadritos pequeños, hay en


el grande?
2. ¿Cuántos cuadritos comprende la
imagen?
3. ¿Cuál es la relación que existe entre el
número de cuadrados de la imagen
respecto al número total de cuadrados
del cuadro?

3.3.2 Situación didáctica 2


Calorías = Gramos

Si al comer 125 gramos de cereal se consumen 500 calorías, ¿cuántos gramos de


cereal se deben comer para consumir 750 calorías?

3.3.3 Situación didáctica 3


La proporción aplica a la oferta

Lorena fue a comprar de litro de helado. En la heladería se encuentra el siguiente


cartel:

¿Cuánto deberá pagar Lorena por los de litro de helado?

34
3.3.4 Situación didáctica 4
Reparto equitativo

Seis amigos se reunieron para repartirse de manera equitativa 4 barras de


chocolate, que tienen la siguiente forma:

¿Qué parte le corresponde a cada uno?:

3.3.5 Situación didáctica 5


Engañosas ofertas

Marco fue a Grand Plaza a comprar unos pantalones, que tienen un costo
aproximado de $430, el pantalón se encuentra en 3 tiendas, en la primera opción
tiene una rebaja de 60%, en la segunda un descuento del 50% y se le adiciona el
15% de descuento y en la tercera el pantalón tiene una rebaja del 40% más el
30% adicional de descuento.
¿En cuál de las tres opciones a Marco le costará menos? Justifica tu respuesta

3.3.6 Situación didáctica 6


El ofertón

En un comercio ofrecen tres unidades de un producto al precio de dos, pagando


$8.30. En otro comercio, pagando $11.20 que es el precio de tres unidades del

35
mismo producto nos dan cuatro unidades. ¿En cuál de los dos comercios es más
barato dicho producto?

3.3.7 Situación didáctica 7


Cobro correcto

Amanda quiere regalar a su mamá una esencia de perfume para su cumpleaños y


lo quiere elaborar ella misma. Para esto buscó en Internet distintas opciones para
hacerlo y eligió la siguiente:
Para obtener 120 ml de esencia de perfume se tiene que mezclar 20 ml del
perfume elegido con 100 ml de alcohol.
Amanda fue a una tienda a comprar los ingredientes y se encontró con la siguiente
tabla de precios:
Luego de consultar, el encargado le dijo que podía llevar sólo la cantidad que
necesitara para componer su perfume. Esa fue su decisión: sólo llevó lo necesario.

CANTIDAD DE ML.
INGREDIENTE PRECIO
POR BOTELLA
Alcohol 500 ml $ 200
Perfume 50 ml $ 200

Cuando Amanda fue a la caja para pagar le cobraron $100. ¿Consideras que le
cobraron de manera correcta? ¿Por qué?

3.4 Participantes

La aplicación del proyecto fue concebida en una Escuela Secundaria Oficial del
turno matutino, con un grupo de segundo grado, constituido por 54 estudiantes de
los cuales 26 eran mujeres y 28 son hombres.

36
Con base en lo anterior se decidió trabajar sólo con 20 alumnos, lo cual no quiere
decir que a los 34 restantes no les haya hecho caso, sino que mi prioridad era
observar el trabajo realizado en esos 20 alumnos. Para tal fin se realizó el método
de muestreo que a continuación se describe:

1. Se numeran los alumnos del grupo, del 1 al 54.


2. Se calcula la amplitud del intervalo dividiendo el total de alumnos (54) entre
20 y el resultado se redondea a 3.
3. El primer alumno seleccionado para la muestra será el 3, los siguientes
alumnos se obtendrán sumando 3, hasta llegar a tener 18 alumnos y
posteriormente para completar la muestra representativa, los otros dos
compañeros que hacían falta, se eligieron por números aleatorios
resultando los alumnos con números de lista 1 y 25.

Con el anterior procedimiento se selecccionaron los 20 estudiantes con los que


se integró la muestra representativa del grupo.

El trabajo del resto también fue calificado y evaluado, pero el análisis no se


efectuó con ellos. Para la aplicación se formaron equipos de cuatro estudiantes
cada uno, cada uno de ellos se integraba por un alumno que trabaja y comprende
mejor la asignatura, aquel niño que no trabaja, aquel que habla mucho y el que no
entiende aquello que se le solicita.

Se sabe que cuando los estudiantes se agrupan de manera afín se obtienen


mejores resultados que si el docente le designa su equipo, empero lo que
esperaba es que la persona que entiende más la asignatura me sirviera de
monitor, sin que éste supiera, se argumentó que cada uno fuera el responsable de
sus actos y opiniones, que era un trabajo colaborativo y por tanto al final de la
actividad serían coevaluados por sus compañeros.

De acuerdo con Guitert y Simérez (Maldonado Pérez, 2007) el trabajo colaborativo


es un proceso en el que cada individuo aprende más de lo que aprendería por sí
solo, fruto de interacción de los integrantes del equipo, éste trabajo se da cuando

37
existe una reciprocidad, entre un conjunto de individuos que saben diferenciar y
contrastar sus puntos de vista de tal manera que llegan a generar un proceso de
construcción de conocimiento.

En ocasiones consideramos al trabajo colaborativo un sinónimo del trabajo


cooperativo, o lo concebimos como igualitario, pero esto es erróneo, en el cuadro
1 se presentan las diferencias en entre ambos tipos de trabajo.

Cuadro1

TRABAJO
CARACTERÍSTICAS TRABAJO COOPERATIVO
COLABORATIVO
Acompaña, es un Estructura el trabajo que
El profesor o facilitador
mediador realizará cada grupo
Definida por los
Tarea Asignada por el profesor
miembros del grupo
Cada miembro del grupo se
Responsabilidad por la
Individual y grupal responsabiliza por una parte
tarea
de la tarea
En ocasiones es distribuido
Realizan el trabajo por el profesor entre los
División del trabajo juntos. Baja división de la miembros del grupo. En otras
labor puede ser distribuido por los
miembros
Entrelazadas. Requieren
Subtareas Independientes
trabajo conjunto
En conjunto. En ningún
Juntando las partes realizadas
caso corresponderá a la
Proceso de construir el por cada miembro. Sumatoria
suma de esfuerzos o
resultado final de subtareas realizadas
desempeños
individualmente
individuales.
Asumida por el profesor al
Miembros del grupo con estructurar el trabajo de
Responsabilidad por el
el acompañamiento del alguna manera que le hace
aprendizaje
profesor pensar que el grupo
aprenderá
No fundamental, se Básico. Fundamental.
requiere razonamiento, Privilegia la memorización y
Tipo de conocimiento
cuestionamiento y en pocas ocasiones tendrá
discusión cabida el cuestionamiento
Cuadro 1. Características diferenciadoras del trabajo colaborativo y trabajo
cooperativo. FUENTE: Maldonado Pérez (2007)

El propósito de trabajar de esta forma es que se aprenda juntos para crecer y


fortalecer el contenido, así mismo la responsabilidad de cada uno de los
integrantes.

38
Al principio hubo dificultades para poderse integrar, con el paso de las actividades
el trabajo fue más completo. Al inicio querían integrar el equipo únicamente con
sus amigos, pero después se les hizó notar que era necesario que aprendieran a
trabajar en equipo aún con aquellas personas con las que no estaban
acostumbradas a tratar. Es conveniente hacer notar que existió una excepción
con dos señoritas que por más que se les pidió que se integraran a su equipo no
lo hicieron saliendo de la actividad, se hizo todo lo posible por incorporarlas, pero
ellas no se prestaron al trabajo, se platicó con ellas y el orientador y aun así ellas o
no asistían o se aislaban.

3.5 Descripción espacio-temporal


En cuanto el tiempo, cada clase tiene una duración aproximada de 45 minutos,
debido a la actividad física que se efectúa de 10:00 a 10:30 de la mañana, por lo
cual el tiempo está muy limitado para la realización de distintas tareas.

Durante los días de la aplicación de la propuesta que fueron 16 sesiones, los


estudiantes tenían que movilizarse al equipo que se les asignó para que entre
todos le diera solución a los problemas planteados. Uno puede pensar que al
trabajar de manera colaborativa, el problema se resuelve con mayor exactitud y
más rápido, sin embargo esto no sucede siempre, porque cada cabeza es un
mundo y pensamos de distinta manera, así que a la hora de concluir en la
resolución del problema existen diferencias en los puntos de vista.

Se tenía planeado efectuar la presente propuesta del 13 de marzo de 2012 al 24


de abril de 2012, sin embargo no se pudo efectuar en el tiempo propuesto debido,
a la organización de la escuela, presentándose aplicaciones de examen tipo
ENLACE, prioridad a este tipo de reactivos durante la clase (prepararlos),
ceremonias, y suspensión de clase por asistencia a eventos sociales, como
concurso de oratoria y declamación efectuados en la institución.

39
3.6 Propuesta de intervención
Se diseñó una secuencia didáctica, entendida como un conjunto de actividades
ordenadas, estructuradas y articuladas para la consecución de los objetivos
educativos (Zabala, 1997 p. 16).

El diseño original de este proyecto de intervención se realizó de acuerdo al


método ABP previamente expuesto y teniendo como guía las competencias.
Posteriormente las condiciones de trabajo en el grupo de alumnos seleccionados
hizo necesario que se modificara la estrategia didáctica del método ABP a las
situaciones didácticas.

PLANIFICACIÓN DIDÁCTICA

ASIGNATURA: GRADO:
GRUPOS: “1” Y “2” BLOQUE: I
MATEMÁTICAS II SEGUNDO

APRENDIZAJES ESPERADOS:
COMPETENCIAS A
 Resuelve problemas de proporcionalidad directa DESARROLLAR:
del tipo “valor faltante”, en los que la razón interna
o externa es un número fraccionario  Resolver problemas de manera
 Resuelve problemas que implican el cálculo de autónoma
porcentajes o de cualquier término de la relación:  Comunicar información
Porcentaje = cantidad base × tasa. Inclusive matemática
problemas que requieren de procedimientos  Validar procedimientos y
recursivos. resultados
 Identifica, interpreta y expresa relaciones de  Manejar técnicas eficientemente
proporcionalidad directa o inversa,
algebraicamente o mediante tablas y gráficas.
¿Qué quiero que
Tema
EJE

¿Cómo lo voy a hacer? mis alumnos


aprendan?

Se inicia con el dictado del cálculo mental.


PROPORCION

Como primera actividad se hace un diagnóstico a los alumnos y


alumnas sobre el manejo y utilización de las razones
Cálculo mental
MI

Para ello de forma individual darán respuesta al siguiente problema,


se dará un tiempo determinado de 10 minutos, posterior a ello se
reunirán por equipos de 4 personas para comentar y discutir sus
respuestas

40
.

En cierta ocasión Carolina fue de compras a una tienda de


artesanías ubicada en el centro de Toluca, ahí encontró
unos cuadros muy bonitos, los observó detenidamente
comprendiendo que los habían hecho sobre una cuadrícula,
he aquí los cuadros Noción de razón, de
proporción

 ¿Cuántos cuadritos
pequeños, hay en el
grande?
 ¿Cuántos cuadritos
comprende la imagen?
 ¿Cuál es la relación
que existe entre el
número de cuadrados Porcentaje
de la imagen respecto
PROPORCIONALIDAD Y FUNCIONES

al número total de
MANEJO DE LA INFORMACIÓN

cuadrados del cuadro?

En plenaria se abordará lo realizado por los equipos y un integrante


de equipo pasará a comentar sus resultados. Se introduce el
concepto de razón y proporción.

Una vez finalizada la explicación de razón, de manera individual, los


alumnos construirán 3 rectángulos, cada uno con las siguientes
medidas:
a) 10.5 cm x 6 cm;
b) 3.5 cm x 2 cm; Trazar un
c) 4 cm x 2 cm rectángulo

En cada rectángulo se trazarán las diagonales.

Una vez trazadas, se colocarán los rectángulos a y b de tal manera


que coincidieran sus diagonales

41
Luego, a los rectángulos a y b, se anexa con el mismo
procedimiento el rectángulo C. Los alumnos observarán que
sucede.
En plenaria se discutirá sobre lo sucedido con los rectángulos, si
existieron inconvenientes o no.

Posteriormente, se solicita que dibujen y recorten rectángulos con


las siguientes medidas:

6 cm x 3 cm; 4 cm x 14 cm;
5 cm x 10 cm; 7 cm x 2 cm;
5 cm x 3 cm, 12.5 cm x 4.5 cm.

Y realicen los pasos de los primeros rectángulos. Para dar


respuesta a la pregunta

¿Con cuáles parejas de rectángulos puede hacerse algo parecido a


los rectángulos anteriores?

Justifiquen por medio de un escrito la respuesta.

Se inicia la clase, con el dictado de cálculo mental.

En el pizarrón se coloca un cartel, con una sola dimensión y a los


alumnos se les da a conocer el problema que tendrán que resolver:

Martín fue a una copiadora para reducir una fotografía con la


PROPORCIONALIDAD Y FUNCIONES

medida indicada a continuación:


MANEJO DE LA INFORMACIÓN

Aplicación de la
proporción

Al recibir la copia, se dio cuenta que la foto ( copia) medía de


ancho 6 cm

1- ¿Cuál fue el factor de reducción que aplicó el encargado de


las copias?
2- ¿Cuánto mide de largo el original, si en la copia este lado
mide 15 cm?

42
3- Queremos que la fotografía se amplíe al tamaño de un cartel
que debe medir 45 cm de largo y 18 cm de ancho ¿Cuál es
su factor de proporcionalidad?
4- ¿Qué característica debe tener el factor de proporcionalidad
cuando sirve para ampliar una figura?, ¿y para reducirla?

Posteriormente se plantea a los estudiantes situaciones


problemáticas de tipo geométrico y de la vida diaria que involucren
magnitudes que presenten variaciones proporcionales directas.
Resuelven diversas situaciones problemáticas de la vida diaria cuya
solución se obtendría de la aplicación del concepto de
proporcionalidad directa

Para iniciar la clase, los alumnos se reunirán en los equipos en los


que han estado trabajando y se les dictaran los siguientes
problemas:
PROPORCIONALIDAD Y FUNCIONES
MANEJO DE LA INFORMACIÓN

 Si al comer 125 gramos de cereal se consumen 500


calorías, ¿cuántos gramos de cereal se deben comer para
consumir 750 calorías?

 Si sabemos que un automóvil puede recorrer


aproximadamente 280 km con 20 litros de gasolina,
¿cuántos litros necesita para recorrer 84 km? ¿Cuántos
kilómetros podrías recorrer con un litro de gasolina?

Una vez que los estudiantes dieron respuesta a los problemas, se


solicita que pase un integrante de cada equipo al pizarrón a
presentar su proceso, el docente observará sus explicaciones y por
último se realizará una puesta en común para formalizar el tema

La clase comienza con el dictado del cálculo mental

Al finalizar el cálculo, se plantea al grupo el siguiente problema:

 Para festejar el cumpleaños de Pamela, Mauricio y su


hermana Jessica la invitaron al cine con un grupo de
amigos.
En la taquilla, Mauricio pago el total de $360.00 por los
boletos de las nueve personas que entraron (no hubo
descuentos de ningún tipo).

43
¿Cuánto pagó por persona? ¿Cuánto le costaron los boletos
de él, su hermana y Pamela?

Mientras esperaban a que empezará la película fueron a la


dulcería, donde Jessica invitó un refresco a cada uno; por
los nueve refresco le cobraron $135.00. Para el inicio de la
función algunos quisieron otro refresco y Jessica les dio
$45.00 para comprarlos. Y al terminar la película, pagó
$75.00 por los otros refrescos más. ¿Cuántos del grupo
tomaron refrescos al inicio y al fin de la película,
respectivamente? (todos estos refrescos fueron del mismo
precio)
PROPORCIONALIDAD Y FUNCIONES
MANEJO DE LA INFORMACIÓN

Una vez que los estudiantes dieron respuesta a los problemas, se


solicita que pase un integrante de cada equipo al pizarrón a
presentar su proceso, el docente observará sus explicaciones y por
último se realizará una puesta en común para formalizar el tema

 Seis amigos se reunieron para repartirse de manera Repartición


equitativa 4 barras de chocolate, que tienen la siguiente equitativa
forma:

¿Qué parte le corresponde a cada uno?:

Posteriormente se solicita que se reúnan en equipos de cuatro


personas y comparen además de argumentar su respuesta. Una
vez que dialogaron entre ellos, se realiza la plenaria por lo cual, se
solicita a cuatro niños pasen al frente a resolver el problema en el
pizarrón para observar cómo resolvieron el problema y
posteriormente formalizar

Se solicita resuelvan de manera individual los siguientes problemas,


posteriormente se reunieran en equipos para debatir resultados.

Razón
 Lorena fue a comprar 34 de litro de helado. En la heladería se
encuentra el siguiente cartel:

44
Identificación de una
proporcionalidad
directa

3
¿Cuánto deberá pagar Lorena por los 4
de litro de helado?

Una vez que culminen el problema del ofertón se le solicita a los


PROPORCIONALIDAD Y FUNCIONES

alumnos den respuesta a los siguientes:


MANEJO DE LA INFORMACIÓN

 Juan tomó un taxi, se puso a platicar con el taxista y le


pregunto cómo funcionaba el taxímetro. El taxista le dijo:
“cuando sube un pasajero, enciendo el taxímetro, el cual
marca $4.00 por el banderazo y aumenta $0.95 cada 100
metros” ¿cuánto pagará si lo llevarán a 600 m del lugar de
origen?

 El agua es un compuesto químico, cuya fórmula es H2O, es


decir, cada molécula de agua contiene dos átomos de
hidrógeno y uno de oxígeno. El agua forma una capa Proporción en
alrededor de la tierra que recibe el nombre de Hidrosfera. diversos contextos
La hidrosfera está conformada por el agua salada de los
océanos, el agua dulce de los ríos, lagos y corrientes
subterráneas, por hielos, y por el vapor de agua que hay en
la atmósfera. El total de agua que hay en la tierra, es de
1300 millones de Km3, distribuidas así: 97% agua salada,
3% agua dulce, 77,5% hielo polar y glaciares, agua
continental, superficial y atmosférica 0,5%, aguas
subterráneas 22%, lagos y zonas húmedas 92%, ríos 1%,
atmósfera 7%. ¿Cuantos Km3 de agua salada hay en la
tierra?

¿Qué aceleración se producirá para mover una piedra si su


masa es de 18kg y sobre esta se le aplica una fuerza de
45N?

En equipos resuelvan el siguiente problema, pueden usar su


calculadora.

Analicen la información de la siguiente tabla y contesten:

45
¿Qué alimento de la lista es más rico en carbohidratos, cuál en
proteínas y cuál en lípidos?
PROPORCIONALIDAD Y FUNCIONES
MANEJO DE LA INFORMACIÓN

Es importante que se analicen los procedimientos empleados,


identificando las ventajas de cada uno. Por la cantidad de
comparaciones de razones, en este caso es interesante cuidar la
economía del tiempo, obtener con la calculadora las cantidades de
carbohidratos, proteínas y lípidos por cada gramo de alimento y
posteriormente comparar los decimales obtenidos, podría resultar
un camino práctico, pero esto, por supuesto, hay que ver si se le
ocurre a los alumnos

Se evaluará que los estudiantes se hayan formado en equipos y


que en la plenaria hayan discutido como dar solución al problema,
además de vincular esto con el bloque de nutrición en ciencias, por
lo cual la devolución se efectuará una vez que los estudiantes
hayan logrado comprender el contenido matemático (proporción)

En la sesión se trabajara con las promociones que las tiendas


departamentales ofrecen a la comunidad como momentos para
invertir mejor su dinero.

Se comienza con el primer problema:

Marco fue a Grand Plaza a comprar unos pantalones, que


tienen un costo aproximado de $430, el pantalón se
encuentra en 3 tiendas, en la primera opción tiene una rebaja
de 60%, en la segunda un descuento del 50% y se le
adiciona el 15% de descuento y en la tercera el pantalón
tiene una rebaja del 40% más el 30% adicional de descuento.
¿En cuál de las tres opciones a Marco le costará menos?
Justifica tu respuesta

46
Algunos de los alumnos comenzaran a dar respuesta, sin embargo
se solicita realicen todas las operaciones.

Una vez concluido el problema se realiza una puesta en común


para observar los distintos puntos de vista.

Posteriormente se trabaja con otro problema:

En un comercio ofrecen tres unidades de un producto al precio de


dos, pagando $8.30. En otro comercio, pagando $11.20 que es el
precio de tres unidades del mismo producto nos dan cuatro
unidades. ¿En cuál de los dos comercios es más barato dicho
producto?

Los alumnos tienen más ideas de cómo obtener el resultado así que
emplearan diversas estrategias para dar solución al problema
PROPORCIONALIDAD Y FUNCIONES

Cálculo mental
MANEJO DE LA INFORMACIÓN

Se dictaran 10 operaciones
El alumno solo debe anotar el resultado Proporción para
cada gramo
Amanda quiere regalar a su mamá una esencia de perfume para su
cumpleaños y lo quiere elaborar ella misma. Para esto buscó en
Internet distintas opciones para hacerlo y eligió la siguiente:
Para obtener 120 ml de esencia de perfume se tiene que mezclar
20 ml del perfume elegido con 100 ml de alcohol.
Amanda fue a una tienda a comprar los ingredientes y se encontró
con la siguiente tabla de precios:
Luego de consultar, el encargado le dijo que podía llevar sólo la
cantidad que necesitara para componer su perfume. Esa fue su
decisión: sólo llevó lo necesario.

Cuando Amanda fue a la caja para pagar le cobraron $100.


¿Consideras que le cobraron de manera correcta? ¿Por qué?
En todo momento
hay matemáticas
En parejas, leerán la siguiente información y resolverán las
siguientes preguntas.

Con 5 kg de maíz se hacen 3 kg de harina. Con 2 kg de harina se


hacen 5 kg de masa. Finalmente con 10 kg de masa se hacen 7 kg
de tortillas.

47
a) Con base a la información anterior calculen la cantidad de
tortillas que se produce con 20 kg de maíz.
b) Determinen la cantidad de maíz necesaria para 35 kg de
tortillas. Numero
c) Calculen la cantidad de harina que se Produce con 1 kg de proporcional
maíz.
d) Encuentren el factor de proporcionalidad.
PROPORCIONALIDAD Y FUNCIONES
MANEJO DE LA INFORMACIÓN

De acuerdo a lo anterior completen las siguientes expresiones


Una vez realizada la puesta los alumnos de manera individual Factor desconocido
completarán el siguiente

h=
t=
__________ M=
__________
m __________ h
M
m= h=
M=
__________ h __________
__________ t
M
t = __________ m
m = __________ t

Utilicen las expresiones anteriores para calcular:

A) La cantidad de tortillas que se produce con 50 kg de maíz


B) La cantidad de maíz que se requiere para 42 kg de tortillas

Una vez que culminan el problema anterior se les da:

Se calcula que se necesitan 20 litros de agua diarios para cada 15


niños que van a una excursión. ¿Cuántos litros se necesitan si 45
niños salen durante 7 días?

48
Al organizar otra excursión el responsable llevó 60 niños y
transportó 420 litros de agua ¿Cuántos días podrá durar la
excursión, si se conserva el promedio de consumo de agua por
cada niño?

Con el objetivo de definir el concepto de proporcionalidad


planteamos un conjunto de situaciones diversas para que decidan si
hay proporcionalidad o no la hay.

Se reparte de manera individual, una vez que terminar de contestar,


se reunirán para cotejar sus resultados entregando una sola
quiniela por equipo.
Una vez que en equipo se dio solución se realiza una puesta en
común, en la cual se dará un mayor énfasis a los argumentos que
se utilizan para defender su respuesta

QUINIELA DE LA PROPORCIONALIDAD

Colocamos una cruz en el 1 si creemos que hay proporcionalidad


claramente, en el 2 si no la hay y en la X si tenemos dudas o no
está muy claro

SITUACIONES PARA ANALIZAR 1 X 2


1.- La cantidad de naranjas y su precio
2.- La edad y la altura de una persona
3.- El número de aprobados y las horas de estudio
4.- La velocidad de un coche y el espacio recorrido
5.- La velocidad de un coche y el tiempo invertido
6.- La cantidad de agua que sale por un grifo y el
tiempo necesario para llenar un recipiente
7.- Precio de una llamada por teléfono y tiempo que
hemos estado hablando
8.- Grosor de un árbol y su edad
9.- Número de pintores y tiempo que necesitan para
hacer pintar un piso
10.- Número de habitantes y consumo de agua de
un municipio
11.- La longitud de la circunferencia y su radio
12.- Número de gallinas y tiempo que dura el pienso
en una granja
13.- Dinero invertido en la plantilla de un equipo de
futbol y puesto de la clasificación
14.- El peso de un melón y su precio
15.-La talla de una camisa y su precio

La actividad consiste en que cada equipo investiga todo lo


relacionado con proporcionalidad, una vez que cada uno tiene su
investigación, realiza una paráfrasis de la investigación y con sus

49
propias palabras nos comparte el concepto o la idea que tienen con
la proporcionalidad, qué es, en qué consiste, en qué momento se
lleva a cabo y claro qué relación establece con nuestra vida diaria,
todo ello deberá ser manifestado con las palabras de cada uno, así
el código que ellos manejan será el mismo y se entenderá mejor el
concepto entre pares.
Una vez que tiene su concepto de proporcionalidad ellos van a
crear 5 problemas del tema con relación a la cotidianeidad

Autoevaluación Coevaluación Heteroevaluaciòn


Por medio del
A través de una rúbrica, se
portafolio, un
evaluará el cómo resolvieron los
Tipos de evaluación Con las notas alumno presentará
problemas
realizadas en su su portafolio a otro
Por medio de una escala de
portafolio y el evaluador hará
rango, en donde por equipos,
sus observaciones
cada compañero evalúa a otro
en una hoja

De acuerdo con Ahumada (2005) (Ahumada, 2005), una rúbrica permite


especificar con claridad lo que se espera que los alumnos logren en su
desempeño, además de evaluar los problemas, permitiendo describir de manera
cualitativa los distintos niveles de logro alcanzados o deseados

50
Capítulo 4.- Descripción y análisis de resultados

En este capítulo se incluyen las situaciones didácticas aplicadas que se diseñaron


para este proyecto de intervención. En cada una de ellas se hace la descripción
que cuenta lo que dice el problema a trabajar, posteriormente se realiza el análisis
de los resultados obtenidos, valorando así los procedimientos utilizados por los
alumnos para resolver la situación didáctica propuesta. Lo anterior nos permite
conocer lo sucedido durante la sesión para llegar a interpretar el cómo y el por qué
de los procedimientos, así como la validación de los resultados.

Para llevar a cabo la descripción y análisis de resultados, éstos se han agrupado


en dos categorías de análisis, con tres subcategorías.
 El alumno establece la razón geométrica como un antecedente de una
proporción.
o El alumno reconoce el concepto de razón geométrica
 Cómo conceptualiza el alumno a la razón
 Como comparador
 Como fracción
 Como porcentaje
 El alumno conoce el concepto de proporcionalidad directa
o Cómo conceptualiza el alumno a la proporcionalidad
 El alumno conoce el concepto de proporcionalidad directa
 El alumno conoce los distintos procedimientos para resolver
una situación de proporcionalidad

4.1 Descripción de resultados


El grupo como ya lo indiqué era muy grande causando dificultad tanto a los
docentes como a los estudiantes debido a que no se les brindaba la misma
atención a todos. En ocasiones no se podía revisar el trabajo o las tareas al grupo.
Las alternativas que se tomaron fueron dejar las tareas en hojas blancas para que
éstas pudieran ser entregadas y revisadas extraclase. De otro modo no se podría

51
hacer una revisión precisa de las tareas, mientras en el aula lo que siempre se
hizo, previa planeación, fue comenzar con algún problema dictado.

Como una forma de trabajo en el aula lo que hago es lo siguiente: les doy tiempo
para que ellos den respuesta al problema. Después solicito que intercambien sus
cuadernos para calificarse. En el momento que dos o tres compañeros pasan a
dar solución al problema, observó si existen diferencias de opinión y realizo la
puesta en común, en donde cada uno expone sus puntos de vista y llegamos a la
solución del problema. Si el tiempo da para otro problema se les da, ya sea para
trabajarlo en la clase o en casa. Siempre que uno termina antes del tiempo
destinado, reviso el problema y pongo en duda el resultado. El alumno revisa
nuevamente su resultado para comprobar que su respuesta es correcta. Durante
ese tiempo los alumnos que están a la espera de los que ya terminaron al no
encontrar quién les dé la respuesta, tienen que trabajar en la búsqueda de la
solución por sus propios medios o apoyados por aquel que terminó.

Sin embargo hay ocasiones en las cuales los estudiantes se preocupan más por
obtener una buena nota que por aprender. Algunos estudiantes ayudan a sus
compañeros a corregir los resultados por lo que todos muestran el resultado
correcto. Cuando llego a observar lo anterior, procuro pasar a los distintos lugares
y encerrar los resultados para que no los alteren y estar en condiciones de
hacerlos reflexionar sobre la “solución” que encontraron.

4.1.1 Descripción de resultados por situación didáctica aplicada


Con base en el esquema de intervención a continuación se presenta la aplicación
de cada una de las situaciones didácticas propuestas.

4.1.1.1 Situación didáctica 1


La cuadrícula
En cierta ocasión Carolina fue de compras a una tienda de artesanías ubicada en
el centro de Toluca, ahí encontró unos cuadros muy bonitos, los observó

52
detenidamente comprendiendo que los habían hecho sobre una cuadrícula, he
aquí los cuadros
1. ¿Cuántos cuadritos pequeños, hay en
el grande?
2. ¿Cuántos cuadritos comprende la
imagen?
3. ¿Cuál es la relación que existe entre el
número de cuadrados de la imagen
respecto al número total de cuadrados
del cuadro?

4.1.1.2 Situación didáctica 2


Calorías = Gramos
Si al comer 125 gramos de cereal se consumen 500 calorías, ¿cuántos gramos de
cereal se deben comer para consumir 750 calorías?

Una vez que los alumnos tienen el problema, empiezan a resolverlo de manera
individual, después de unos minutos dentro de su equipo discute el resultado
obtenido, con el fin de conocer la forma en que cada integrante resuelve el
problema.

2.1.1.3 Situación didáctica 3


La proporción aplica a la oferta

Lorena fue a comprar de litro de helado. En la heladería se encuentra el siguiente


cartel:

53
¿Cuánto deberá pagar Lorena por los ¾ de litro de helado?

4.1.1.4 Situación didáctica 4


Reparto equitativo
Seis amigos se reunieron para repartirse de manera equitativa 4 barras de
chocolate, que tienen la siguiente forma:

¿Qué parte le corresponde a cada uno?:

Cuando se les dio el problema a los estudiantes la actitud que tomaron fue positiva
al creer que era algo tan sencillo, he hicieron comentarios de “¡Ay! maestra esta re
fácil” “ponga otro problema” “ya lo terminamos”, entre otras, pero en el momento
en que solicitaba que todo el equipo tenía que calificarse al mismo tiempo y que
uno de los integrantes tenía que explicar cómo lo resolvieron, observaron que
cada uno tenía un criterio distinto.

4.1.1.5 Situación didáctica 5


Engañosas ofertas
Marco fue a Grand Plaza a comprar unos pantalones, que tienen un costo
aproximado de $430, el pantalón se encuentra en 3 tiendas, en la primera opción
tiene una rebaja de 60%, en la segunda un descuento del 50% y se le adiciona el
15% de descuento y en la tercera el pantalón tiene una rebaja del 40% más el
30% adicional de descuento.

¿En cuál de las tres opciones a Marco le costará menos? Justifica tu respuesta

54
Al terminar de leer la situación didáctica, hubo alumnos que sin realizar ningún
procedimiento escrito, dijeron que la opción tres era la correcta, por lo que se les
solicitó que realizarán lo necesario para fundamentar su respuesta.

En la 3:
“Datos:
$430 pantalón
1. 60%
2. 50% + 15% = 65%
3. 40% + 30% = 70%”

4.1.1.6 Situación didáctica 6


El ofertón

En un comercio ofrecen tres unidades de un producto al precio de dos, pagando


$8.30. En otro comercio, pagando $11.20 que es el precio de tres unidades del
mismo producto nos dan cuatro unidades. ¿En cuál de los dos comercios es más
barato dicho producto?

4.1.1.7 Situación didáctica 7


Cobro correcto

Amanda quiere regalar a su mamá una esencia de perfume para su cumpleaños y


lo quiere elaborar ella misma. Para esto buscó en Internet distintas opciones para
hacerlo y eligió la siguiente:

Para obtener 120 ml de esencia de perfume se tiene que mezclar 20 ml del


perfume elegido con 100 ml de alcohol.

55
Amanda fue a una tienda a comprar los ingredientes y se encontró con la siguiente
tabla de precios:

Luego de consultar, el encargado le dijo que podía llevar sólo la cantidad que
necesitara para componer su perfume. Esa fue su decisión: sólo llevó lo necesario.

Cuando Amanda fue a la caja para pagar le cobraron $100. ¿Consideras que le
cobraron de manera correcta? ¿Por qué?

4.1.1.8 Situación didáctica 8


La proporcionalidad en la vida cotidiana

La actividad consistió en que cada equipo debía investigar todo lo relacionado con
proporcionalidad. Una vez que cada uno tiene su investigación, realiza una
paráfrasis de la investigación y con sus propias palabras nos comparte el concepto
o la idea que tienen de la proporcionalidad, qué es, en qué consiste, en qué
momento se lleva a cabo y claro qué relación se establece con nuestra vida diaria.
De manera que se pudiera establecer el diálogo entre pares, toda vez que ellos
utilizan un mismo código por lo que se entenderá mejor el concepto de
proporcionalidad.

Una vez que ellos tienen su concepto de proporcionalidad, procedieron a elaborar


5 problemas del tema con relación a la cotidianeidad.

56
Por falta de tiempo, el equipo que a mi parecer fue el que tuvo mejor desempeño
en el tema porque relacionó más los problemas elaborados con la vida cotidiana,
no pudo aplicar sus problemas al grupo.
Los problemas planteados por el equipo fueron los siguientes:

 ¿Qué distancia se puede recorrer en una bicicleta en 25 minutos con una


rapidez de 55 km/h?

 El lunes hice un examen de 75 preguntas, la miss nos dijo que quien tuviera
las 75 respuestas bien tenía 10 de calificación ¿cuánto saque si tuve 60
aciertos?

 Pepe fue a la tienda y pidió 6 kilogramos de arroz, pero aparte quería 500
gramos de éste. Si por los 6 kilogramos le cobraron $150, ¿Cuánto le
cobrarán por los 500 gramos?

 Andrea tarda una hora en recorrer 150 km ¿cuántos kilómetros recorrerá en


150 minutos?

 Vea un programa de televisión contabilice el tiempo promedio de un


comercial. Cuente el número de comerciales en el programa que ve. ¿Si
usted ve 4 Horas de televisión que porcentaje corresponde a comerciales?
¿Cuántos comerciales ve usted en promedio por 4 horas de televisión?

Para concluir se les preguntó a los alumnos del grupo cómo consideraban que
fueron los problemas planteados durante toda la secuencia. Los resultados de sus
respuestas se resumen en lo siguiente: ocho alumnos (15%) opinaron que les
resultó interesante; 14 alumnos (25%) dijeron que los problemas eran
comprensibles y a 32 alumnos (60%) les parecieron complejos.

57
4.1.2 Análisis de resultados
4.1.2.1 Situación didáctica 1
La cuadrícula
El alumno realizó lo que se esperaba, pues era solo que ellos se aproximarán al
concepto de razón geométrica. En la hoja que se dio a los alumnos se plantearon
las preguntas a resolver con la figura. Ellos fueron quienes realizaron lo necesario
para dar respuesta a las interrogantes.

Cada alumno del grupo trató de dar respuesta a la pregunta ¿cuántos cuadritos
pequeños hay en el grande?, y procedió a contarlos. Algunos contaron cuadrito
por cuadrito, otros contaron la primera fila vertical y horizontal para después
multiplicar. Todos llegaron al resultado correcto que es 100.

Para la segunda pregunta ¿Cuántos cuadritos comprende la imagen?, los alumnos


debían contar el número de cuadritos que tiene la figura. Algunos alumnos
procedieron a contar uno por uno, otros colocaron una señal en cada cuadrito para
no volverlo a contar. Llegando al resultado correcto, sin embargo aunque parece
una respuesta obvia hubo dos personas que no contestaron correctamente a la
pregunta. Al igual que en la pregunta anterior sólo tenían que contar el número de
cuadritos que constituye la figura. La sumatoria al final fue la misma, excepto para
esos dos alumnos, uno de ellos contó tres cuadritos más y el otro contó uno
menos.
En la tercera pregunta ¿Cuál es la relación que existe entre el número de
cuadritos de la imagen respecto al número total de cuadritos que contiene cada
cuadro? Se encontraron diversas respuestas, entre ellas, respondían que había 28
cuadritos coloreados de 100 en la primera figura, para la segunda que eran 48 de
100, otros solamente escribían el número de cuadritos sombreados en cada caso
y otros expresaban el resultado como porcentaje, 28% y 48% respectivamente.
En esta situación pude observar que, cinco de los veinte alumnos (25%) del total
de la muestra, no tiene aún bien asimilado el concepto de razón geométrica, quizá
porque no han conceptualizado que la razón geométrica es el cociente de dos

58
cantidades. Tal vez esto se deba a que como docentes partimos de que los
alumnos ya tienen el antecedente de dicho concepto y en ocasiones sólo nos
remitimos a utilizar los conceptos sin asegurarnos de que cuenten con los
antecedentes necesarios y suficientes. La tercera pregunta lleva implícita la
aplicación de la definición de razón geométrica.

Como una síntesis de los resultados anteriores se puede decir que en la primera
pregunta todos contestaron de manera correcta; en la segunda dos de los veinte
alumnos (10%) dieron un resultado incorrecto, mientras que en la tercera pregunta
15 de los 20 alumnos (75%) dieron una respuesta válida.
El procedimiento empleado por la mayoría de los alumnos (75%) consistió en
expresar la relación entre el número de cuadritos sombreados y el total de
cuadritos como una expresión del tipo: 28 de 100 ó 48 de 100 que se puede
considerar como conocimiento cotidiano que es un buen antecedente del
conocimiento científico.
Dichas respuestas fueron tomadas como validas debido a que una razón
geométrica se lee: “48 es a 100” y se representa como: ó 48:100.
Los conocimientos que los alumnos poseían al empezar a resolver la situación
eran cotidianos, pues desconocían que aquello que realizaron era una razón
geométrica. Al terminar de resolver la situación didáctica se apropiaron de la forma
de leer y expresar una razón geométrica.

En cuanto a las habilidades, los alumnos tienen procedimientos muy marcados,


como es la forma de saber cuántos cuadritos poseía el marco, al contar cuadrito
por cuadrito, mientras que otros alumnos, sin que se solicitara, expresaron el
resultado como un porcentaje.

La actitud, siempre fue proactiva y colaborativa, cuando se requirió, ya que en su


inicio cada uno resolvió el problema y después de unos minutos, se integraron en
equipo para discutir y validar los resultados.

59
Posteriormente expliqué el concepto de razón geométrica e indiqué cómo lo
podemos escribir y leer. Aquí pude observar que ellos identifican la representación
de una razón geométrica como una fracción común por lo que fue necesario
explicarles que éste es otro de los significados que se puede dar a la fracción
común.

Esta situación fue elegida para que los estudiantes comiencen a comprender el
concepto de proporción, la cual es la igualdad de dos razones, donde cada razón
es la comparación entre dos cantidades medibles y se puede representar con una
fracción que se lee “a” es a “b”, lo que se les pedía era comparar los cuadritos
sombreados con respecto al total de cuadritos del cuadrado grande.

4.1.2.2 Situación didáctica 2


Calorías = Gramos

En esta situación didáctica noté un equilibrio entre la utilización de la técnica de


regla de tres y la utilización de la división.
La regla de tres consiste en encontrar la cuarta proporcional cuando se desconoce
uno de los datos de las dos razones que forman una proporción, de tal forma que
ellos aplicaron la propiedad fundamental de las proporciones para encontrar el
dato desconocido. Este procedimiento lo utilizaron 10 alumnos que representan el
50% de la muestra.

Algunos alumnos procedieron de la siguiente forma: observaron que 750 se puede


componer de 500 más su mitad, es decir 250, así que para obtener la solución, es
decir, los gramos requeridos, dividieron 125 entre 2 y al resultado le sumaron 125,
llegando así a la respuesta solicitada. De esta forma procedieron cuatro alumnos
que corresponden al 20% de la muestra

Otros alumnos utilizaron la técnica del valor unitario que consiste en dividir el valor
total entre el número de partes para encontrar el valor de una de ellas, es decir,

60
125 gramos entre 500 calorías encontrando como valor unitario 0.25, esto significa
que cada 0.25 gramos proporciona una caloría. Para encontrar la cantidad de
gramos del cereal necesarios para generar 750 calorías, multiplicamos esta
cantidad por 0.25 para obtener 187.5 gramos de cereal. Este procedimiento lo
utilizaron 6 alumnos que representan el 30% de la muestra.
Los alumnos dieron respuesta al problema donde se les solicita saber cuántos
gramos de cereal se deben consumir para producir 750 calorías, observando que
debían ser más de 125 gramos de cereal pues con ellos se obtenían 500 calorías,
de tal forma que dieron solución de manera aritmética, para ello se fijaron en que
si a 500 se le agrega la mitad (250) se obtienen 750 por lo que razonaron que ha
125 se le debe sumar su mitad (62.5) con lo cual obtuvieron 187.5 como
respuesta. Esta forma de resolver la situación didáctica corresponde a lo que se
conoce como conocimiento cotidiano.

Sabemos que para la proporción directa se debe cumplir con el principio de que si
uno de sus elementos aumenta en una cierta cantidad (o disminuye) el otro
elemento debe aumentar (o disminuir) también. De esta manera los alumnos
lograron dar respuesta, utilizando la propiedad fundamental de las proporciones
por medio de la regla de tres o bien determinando lo que se conoce como valor
unitario.

Los alumnos parten del concepto de razón geométrica. En su mayoría los alumnos
poseen el concepto de proporcionalidad. Conocen la regla de tres simple, en la
cual ellos acomodan los tres datos que se dan a conocer en el problema para
encontrar el cuarto dato que hace falta y así completar la proporción.

El alumno sabe que debe acomodar dichos datos en dos columnas las cuales
deben estar siempre en el mismo orden, si hablamos en este caso de calorías y
gramos, de un lado debe encontrarse los gramos únicamente y del otro las
calorías, esta disposición se puede hacer de manera horizontal o vertical.

61
El alumno pudo disponer los datos de alguna de las siguientes formas:
Gramos Calorías
Gramos 125 X
125 500
Calorías 500 750
X 750

Después multiplicó los datos conocidos 125 por 750 y dividió el producto entre 500
para encontrar el valor de x.

Los alumnos lograron identificar diferentes procedimientos para resolver el


problema que se plantea en la situación didáctica, esto permitió comunicar otras
formas de dar solución a los problemas que después ellos diseñaron.

Los alumnos reaccionan de manera favorable a la solución de problemas cuando


los comprenden y los pueden resolver.

4.1.2.3 Situación didáctica 3


La proporción aplica a la oferta

La situación didáctica planteada arrojó varios procedimientos que dieron lugar a


distintas respuestas diferentes a la que yo esperaba que obtuvieran que era
$37.50, esto porque de litro equivalen a 0.75 que expresado como porcentaje se
escribe en la forma 75% y el 75% de $50 es $37.50.

Esta respuesta es la que se esperaba que ellos obtuvieran, porque la oferta era
$50 por un litro de helado, de tal forma que a partir de esta información se debería
pagar $37.50.
Forma 1

50 100
x 75
50 75 3750
𝑥 𝑥 75
100 100

62
Una vez obtenido el resultado anterior, se les haría notar a los alumnos que la lista
de precios no aclaraba que la oferta de $50 era únicamente aplicable cuando se
compraba un litro, de tal forma que debían obtener el precio real del litro de helado
para posteriormente encontrar el costo de de litro de halado.
Sin embargo, esto no fue posible porque ellos se adelantaron y encontraron la
respuesta correcta de $45. 15 de los 20 veinte alumnos de la muestra obtuvieron
este resultado.

Los razonamientos que utilizaron para obtener el resultado de $45 fueron los
siguientes:

Forma 3
Forma 2

𝑥 + +
𝑥 +

𝑥 + 5 𝑥
𝑥 5 𝑥 5
𝑥 5

Para 9 de los 20 alumnos de la muestra fue tan fácil sumar el precio del medio litro
y el precio del cuarto litro, dando como resultado el ya mencionado (Forma 2).
6 de los 20 alumnos de la muestra lo que hicieron fue sumar tres veces el precio
de un cuarto de litro de helado para obtener los tres cuartos de helado (Forma 3).

Por lo anterior me vi en la necesidad de cambiar la pregunta que daba continuidad


al problema, para conocer el razonamiento empleado en la obtención del
resultado.

Replanteando las preguntas.


 El señor de la tienda le cobro $45 ¿por qué?
 ¿Por qué Lorena pensaba que le cobrarían $37.50?

63
Las respuestas fueron las siguientes:
“porque pensó que medio litro costaba $25 y que el cuarto de litro costaba $12.50
y como 25 + 12.5 = 37.5, que son tres cuartos de helado”

Todos respondieron que obtuvo los tres cuartos de 50, así dividieron entre cuatro y
multiplicaron por tres, esos fueron sus argumentos.

“Por qué Lorena creyó como costaba $50 el litro entonces si lo divide entre cuatro
y lo multiplica por tres le da $37.50 pero no se dio cuenta que el cuarto cuesta 15
pesos y si lo sumabas o lo multiplicabas por tres te daba $45 pesos”

Y la última pregunta a este problema fue ¿por qué se produjo la confusión y qué
dato deben cambiar para que no exista tal confusión?

R= “porque el litro no costaba $50; deben quitar el letrero de 1 litro a $50”

Otra forma de resolver la situación didáctica, consistió en restar al entero una


cuarta parte, es decir, al precio de un litro de helado $50 se le restó el precio de un
cuarto de litro de helado $15, con lo cual se obtuvo el siguiente resultado.
Forma 4

𝑥 −

𝑥 5 − 5
𝑥 5

Esta manera de dar solución al problema lo presentó únicamente un alumno.


Otra forma de llegar al resultado consiste en dividir la unidad entre cuatro y
multiplicarlo por tres.
Forma 5

5 ÷
5 75

64
Los alumnos recurren a determinar tres cuartas partes del entero, realizando la
división del número que se tomó como unidad entre cuatro y al resultado
multiplicarlo por tres obteniendo así lo que necesitamos pagar, esta forma de
realizarlo fue casi de la mitad (45%) de los alumnos de la muestra, mientras que
tres decimos (30%) de ésta realizaron la suma de un medio más un cuarto para
obtener los tres cuartos que nos solicita el problema, solo una persona (5%)
encontró una tercera forma de resolverlo en donde al precio del litro le restan el
costo de un cuarto dando así el resultado del costo de los tres cuartos, mientras
que una quinta parte (20%) no coincidieron con las respuestas que los otros
equipos obtuvieron pues realizaron la proporción con base en el precio del litro de
helado.

Para obtener el precio de los tres cuartos de helado los alumnos reconocen que
requieren hacer una proporción observando las razones que se tienen. Deben de
identificar cuando un problema tiene que ser resuelto por una proporción
aritmética y cuando por medio de una proporción geométrica.

Conocimientos: utilizan diversos métodos o procedimientos para llegar a la


solución de un problema, saben que la suma de fracciones cuando tienen el
mismo denominador, éste se pasa igual y sólo se opera con los numeradores,
además de utilizar los algoritmos de suma y resta de fracciones.

Capacidades: El alumno es capaz de establecer esquemas en los cuales


podemos distinguir cuando existe una proporción y cuando esta no es funcional.

Actitudes: fue de curiosidad al observar que en un problema podíamos encontrar


tres respuestas correctas de acuerdo con el procedimiento utilizado, sin embargo
uno solo de ellos es válido.

65
4.1.2.4 Situación didáctica 4
Reparto equitativo

Al momento de evaluar los procedimientos y resultados obtenidos con cada uno


de ellos se encontraron las siguientes respuestas:
 Contar el número de divisiones que tiene cada chocolate y dividirlo entre los
seis, al hacerlo de esta manera se dieron cuenta de que a cada amigo le
correspondían 6.3 pedacitos de los 38 totales, con la observación de que no
todos los pedacitos eran iguales.
 Fueron colocando números en representación de los amigos; en tres de las
cuatro barras les corresponde una división y solo es en una de las barras
donde les corresponden dos trozos, sin embargo en algunas barras aún les
sobró, por lo cual procedieron a repartirlas pero sin que fueran pedazos
equitativos.
 Otro equipo omitió las divisiones que los chocolates poseían, así que cada
barra fue repartida de manera fraccionada.
 La respuesta más acertada fue la de un equipo que utilizó colores y barras
de chocolate que tenían 8, 10 y 14 divisiones; las dividieron en mitades, es
decir, que de estas tres barras a cada amigo le corresponde una mitad y la
barra que estaba dividida en sextos le corresponde un sexto a cada amigo,
por lo tanto sumaron un medio más un sexto, obteniendo así cuatro sextos
o dos tercios. Así a cada amigo le corresponden 2/3 de barra.
+
+

Al resolver el problema, los alumnos aún tienen dificultades al concebir como


iguales a todas las porciones de los distintos chocolates, sin observar que no
tienen las mismas dimensiones, por tanto no son congruentes, así la mitad (50%)
de los alumnos observados encuentra que se deben sumar el número total de los
fragmentos de cada chocolate y dividirlo entre los seis amigos que son, dando

66
como resultado que a cada amigo le corresponden seis pedacitos y una tercera
parte de otro pedacito, mientras que un 30% aproximadamente de la muestra
difiere de los anteriores y menciona que a cada amigo le corresponden 3
pedacitos, y el resto de los alumnos (20%), concuerdan que la repartición se debe
hacer en partes iguales, dividiendo así tres de las cuatro barras a la mitad y la
barra sobrante dividiéndola en seis partes, de tal forma que a cada amigo le tocan
de barra.

Dentro del conocimiento cotidiano, se tiene la idea de que repartir significa otorgar
a cada miembro una parte del todo, no importando el tamaño de cada una de las
partes.
El conocimiento científico se manifiesta en este caso cuando se suman dos
fracciones que tienen distinto denominador, los alumnos utilizan diferentes formas
para dar solución a la operación. La fracción expresa un repartidor equitativo en
donde el denominador indica el número de partes iguales en que se divide el
entero y el numerador el número de partes iguales que se toman.

Alumno 1

Repartió chocolate por chocolate

Alumno 2:

67
+
+

Alumno 3:

CHOCOLATE 4

1 = 1 cuadrito a cada quien


1
2 = 1 cuadrito y 8 de cuadrito (.33)
2
3 = 2 cuadritos y 3 de cuadro (.66)
2
4 = 1 cuadrito y 3 de cuadro (.66)

68
Alumno 4:

1 1
R= 2 + 6 es la forma
equitativa de repartir
chocolates

Conocimientos: Identifica los principales elementos y fenómenos del medio físico,


así como su organización, características e interacciones.
Capacidades: Reconoce diversas estrategias para dar solución al problema, de
manera cotidiana además utiliza herramientas aritméticas.
Actitudes: Asombro por observar que su proceso de resolución no era válido, y de
dedicación por mejorar lo ya realizado.

Con el propósito de mostrar evidencia de que a cada uno de los alumnos le tocó
de manera equitativa su correspondiente parte de chocolate se procedió a pedir a
cada uno de los equipos que no tuvieron los logros esperados, se solicitó que en
una hoja de papel, trazaran cuatro rectángulos de las mismas dimensiones,
posteriormente se pidió que realizarán las divisiones solicitadas (claro cada
rectángulo dividido en partes iguales, ya sea en 6, 8, 10 o 14 respectivamente),
una vez hecho lo anterior, los alumnos observaron el procedimiento que habían
realizado y lo compararon con lo que en realidad sucedía y anotaron sus
conclusiones para quedar convencidos que el reparto fue equitativo.

69
4.1.2.5 Situación didáctica 5
Engañosas ofertas

Este problema arrojó los resultados esperados. 16 de los 20 alumnos de la


muestra, tenían la idea de que los porcentajes se suman y por tanto cuando hay
una rebaja y a ésta misma se le añade otra, lo que hacen es sumar los
porcentajes para obtener el descuento total. Así que la respuesta la pueden dar
por intuición, al terminar de escribir el problema. Todos decidieron que la opción
tres era la correcta. Para poder hacer la validación del resultado se les pidió que
lo hicieran por partes y encontraron que las apariencias engañan, pues no siempre
los anuncios publicitarios nos muestran con claridad de cuánto es su descuento,
disfrazando así el precio de un producto para ofrecer al público la idea de que sus
precios son más bajos de lo que en realidad son.

16 de los 20 alumnos de la muestra (80%) a primera instancia dijeron que la


respuesta correcta era la tercera opción, justificando que el 40% + 30% dan como
resultado el 70%, idea errónea pues la suma de estos porcentajes es del 58%,
pues si colocamos por ejemplo que el pantalón tiene un costo del $100 y se le
aplica un descuento del 40%, pagaré entonces $60, sin embargo a este precio le
hacen otro descuento del 30%, dando como resultado que el costo final del
pantalón, una vez aplicado el descuento total, será de $42, y no de $30 pesos
como se tenía pensado.
Mientras que la segunda opción (10%) fue elegida por dos alumnos que
encontraron que el 58% es el descuento total de la prenda y por tanto la opción
que más conviene es la primera, que solo dos alumnos eligieron, pues te
descuentan el 60%, pagando así el 40% del total del precio del pantalón.
Los alumnos parten de la idea de que adición significa suma y esto es correcto
cuando se trata de números naturales. En el caso de los enteros, se debe
considerar el signo, por lo que se trata de una suma algebraica, así que llegaron a
la conclusión de que para adicionar un descuento a otro se tienen que multiplicar
los dos descuentos en lugar de sumarlos.

70
Conocimientos: identifica como obtener un porcentaje, y conoce que se multiplican
los porcentajes aplicados a un producto, no se suman. Además de identificar que
el porcentaje es una forma de interpretar a una fracción.
Capacidades Se observa que hay alumnos que están en una etapa inicial de
desarrollo de la habilidad para interpretar y expresar con claridad y precisión
informaciones, datos y argumentaciones, lo que aumenta la posibilidad de
continuar en el camino de incrementar y consolidar esta habilidad para estar en
condiciones de resolver situaciones cotidianas como la anteriormente estudiada.
Actitudes De reflexión al darse cuenta que las ofertas que presentan los anuncios
publicitarios no siempre son claras, sino que más bien resultan engañosas y
constituyen el gancho perfecto para adquirir bienes, o productos. Así que el
alumno se interesa por saber si en realidad la oferta es tal o nos induce a tomar
una decisión equivocada.

4.1.2.6 Situación didáctica 6


El ofertón

De manera intuitiva, gran parte de las personas pensamos que entre más artículos
nos den por menos dinero la oferta es válida, sin embrago en muchas de las
ocasiones no es así, nos dan la idea de que están de liquidación, que es la
semana de las ofertas, pero qué tan verdadero es todo ello, en este caso el 60%
se fue por la opción incorrecta y el resto (40%) acertó a la respuesta.

2.7 6 2.8 0
3 8.3 0 4 1 1.2 0
2 3 3 2
2 0 0 0
2 0

En las operaciones anteriores se puede observar que el cociente de la segunda


división es mayor que el de la primera y por tanto es mejor la oferta de tres por el

71
precio de dos, cuyo costo unitario es de $2.76, mientras que la oferta de cuatro por
el precio de tres su costo unitario es de $2.80.

Algunas de las respuestas obtenidas por los alumnos son las siguientes:
R= en el primero (3x2) porque cada producto cuesta $2.76 y en el segundo $2.80
“la segunda es mejor opción pues por el mismo precio de tres productos del primer
comercio es igual a cuatro productos del segundo comercio por eso es mejor el
segundo”.
El de $11.20 porque te dan más, cantidad del producto (SIC)

Estas son ofertas típicas de las tiendas, en donde los estudiantes deben de
observar siempre qué es lo que más les conviene y no dejarse guiar por el criterio
de que si les dan más artículos el precio unitario será mejor.
Los alumnos consideran que cuando más producto dan por un valor “inferior”
conviene la compra, pero esto no siempre sucede, para poder saber si esto es
correcto podemos obtener el valor unitario en cada uno de los casos y ver si en
verdad es más barato, además de reconocer que no siempre es algo usual, pues a
veces solo requerimos de un solo artículo y por aprovechar las ofertas compramos
artículos de más.

Conocimientos: Reconoce que es importante analizar las proporciones con base


en los precios sugeridos para la toma de decisiones en la adquisición de algún
producto, además de comprender el uso y la aplicación de ésta.
Capacidades Los alumnos reflexionan acerca de los anuncios publicitarios con
base en las ofertas, debido a que muchas veces éstos son engañosos así que en
el momento de ver una oferta, los alumnos deben ser capaces de analizar y tomar
decisiones.
Actitudes Muestran interés por resolver problemas de este tipo pues consideran
que les es útil en su vida cotidiana, además de ser motivadoras en la toma de
decisiones e involucrarse en la compra de la canasta básica del hogar.

72
4.1.2.7 Situación didáctica 7
Cobro correcto

En un inicio, los resultados encontrados para la situación didáctica no fueron los


esperados, pues no la leyeron adecuadamente. Se hizo una segunda lectura, y se
realizaron preguntas para observar si el problema fue más entendible y por ende
la solución fue más precisa. Efectivamente la respuesta era que a Amanda le
cobraron mal. El problema nos brindó una puesta en común muy importante para
aclarar lo que el problema solicitaba.
Pudimos observar que los alumnos, suelen contestar la pregunta en un instante y
no se detienen a leer correctamente el problema, considerando que con solo hacer
una primera revisión pueden lograr dar respuesta al problema. Esto no siempre es
correcto pues debemos de leer para comprender lo que plantea el problema y así
realizar los procedimientos y cálculos que correspondan para dar solución al
problema.

Para este problema se decidió trabajarlo en equipos con la intención de discutir las
posibles respuestas.
2 de los 5 equipos llegaron a la conclusión de que el costo del perfume seria de
$400, debido a que cada cantidad tanto de esencia como de alcohol tenían un
costo de $200. Por lo tanto le había cobrado mal

Otro equipo dijo que no era válido que ambos productos costarán lo mismo pues
en una botella se ofrecían 500 ml y en el otro sólo 50 ml y la diferencia entre una
botella y otra era mucha por lo tanto el costo de la esencia de perfume debía ser
inferior y en consecuencia el cobro fue incorrecto

Mientras que dos equipo si plantearon la proporción en forma correcta, pues


leyeron que el problema hacía mención a que Amanda podía comprar lo
necesario. Y dijeron que no era correcto el cobro.

73
Durante la puesta en común en el grupo, los alumnos dijeron que el cobro estaba
mal y dieron a conocer el porqué de sus respuestas, debatieron sus
procedimientos y observaron que no era obligatorio comprar toda la botella, así
que podían establecer la proporción y obtener el resultado, además de
cuestionarse por qué querían cobrar más de lo debido.

Conocimientos: los alumnos tienen definido cómo resolver el problema, sólo que
hace falta comprender lo que se lee, obtuvieron la respuesta correcta.
Habilidades: Es necesario que el alumno se esfuerce en comprender cuál es la
dificultad que plantea el problema antes de empezar a realizar operaciones, esto
implica que en ocasiones se deba leer varias veces el enunciado para entender
que es lo que se pide.
Capacidades: Los alumnos pueden resolver problemas de proporcionalidad directa
hace falta discutir para argumentar a favor y en contra de algún procedimiento
para obtener la solución por ello es importante que logre entender en un inicio lo
que se pide en el problema

Actitudes: En un inicio fue muy positiva al creer que la respuesta era tan fácil como
decir sí o no; en el momento de solicitar la justificación, la actitud fue cooperativa y
de respeto durante la puesta en común para lograr entender el planteamiento del
problema.

4.1.3 Análisis de la secuencia didáctica

Se realizó un análisis por medio de una rúbrica (anexo 1) para observar cómo
fueron las situaciones didácticas para los estudiantes.
Como resultado de la aplicación de la rúbrica antes indicada se obtuvieron los
siguientes datos de tipo cualitativo. Lo anterior con el propósito de observar en
dónde aún se presentan dudas para trabajar con ellas y reforzar los conocimientos
adquiridos.

74
En la siguiente gráfica se presentan los rasgos evaluados.

Criterios de evaluación de la secuencia didáctica


18
16
14
12 Muy bien
10 Bien
8
Regular
6
Malo
4
2
0
Planteamiento del Datos Incógnita Hipótesis Procedimientos Resultados Conclusiones
problema

Gráfica 2. Criterios de evaluación de la secuencia didáctica.


Resultados obtenidos de la rúbrica aplicada al grupo en relación con la aplicación
de la secuencia didáctica. Fuente: Datos obtenidos del segundo grado grupo 2

75
Conclusiones

Después de aplicar la secuencia didáctica diseñada para este proyecto se puede


concluir que:
Tomando como base el desarrollo de la secuencia didáctica y la evaluación
cualitativa realizada al grupo puedo decir que las actividades fueron interesantes y
al mismo tiempo complejas. A pesar de ello el propósito planteado fue logrado en
más del 50%. Para mí como docente es de suma importancia tomando en cuenta
que de acuerdo al diagnóstico, a los alumnos no les agradan las matemáticas.

De acuerdo con la rúbrica se puede afirmar que el grupo se encuentra en el nivel


de Bien, al identificar el problema, hacer la lista de los datos que se tiene y al
mismo tiempo reconocer lo que se plantea en el enunciado, presenta los
resultados de forma organizada y llega a conclusiones correctas. En cuanto a los
equipos: Intercambian ideas, colaboran en la resolución de los problemas,
atienden y respetan las opiniones de los demás, registran y sistematizan sus
observaciones.

Al inicio de la puesta en práctica de las situaciones didácticas incluidas en la


secuencia tenía la idea de que el cambiar a los alumnos de lugar se propiciaba la
indisciplina por lo cual nunca me había aventurado a intentarlo. Sin embargo en
esta ocasión lo hice y obtuve resultados satisfactorios. En grupos pequeños se
puede trabajar de manera más personalizada, pero cuando el grupo es muy
numeroso, a los docentes nos preocupa más la disciplina que el contenido a
enseñar.

Ha sido posible observar en el grupo que al trabajar en equipos se obtienen


resultados que son muy fructíferos, pues de esta manera se llegan a encontrar
monitores que auxilian a los alumnos que van un poco desfasados, los ayudan a
entender los problemas y esto apoya para que el docente pueda trabajar con todo

76
el grupo, de manera especial con aquellos que tienen dudas y con mayor razón
cuando los grupos son muy numerosos.

Los alumnos obtuvieron una nueva visión sobre el concepto de proporcionalidad,


en la resolución de problemas pues estaban muy acostumbrados a utilizar
únicamente la regla de tres y encontraron que a partir del valor unitario también
pueden encontrar la solución al problema.

Para los alumnos el planteamiento de problemas que implican el concepto de


proporcionalidad les resultó más atractivo porque cada uno de ellos se refiere al
contexto en el cual se desenvuelven al ir a comprar, al hacer una repartición, entre
otras actividades que les resultan cotidianas.

En la situación didáctica la proporcionalidad en la vida cotidiana me resulto


altamente satisfactoria debido a que las explicaciones que los estudiantes
plasmaron en sus trabajos, me permitieron observar el grado de dominio en el
tema.

Es muy importante propiciar que los alumnos adquieran el lenguaje matemático, lo


empleen y lo apliquen apropiadamente en las diversas situaciones que se le
planteen. Esto porque en ocasiones se les dificulta la comprensión de un
enunciado en el que se les plantea un problema. Al tratar el concepto de razón los
alumnos la concebían sólo como una relación entre cantidades sin que les
quedara claro en qué consistía dicha relación establecida entre dos cantidades
medibles.

77
Sugerencias
Por lo anterior sugiero que cuando se trabaje el concepto de proporcionalidad se
utilice material concreto, como una preparación para recurrir gradualmente al
pensamiento abstracto y a la vez hacer que su aprendizaje sea más significativo.

En igual forma sugiero también que al término de los problemas inventados por los
alumnos, se realice un pequeño really por equipos. Esto con el propósito de
consolidar el aprendizaje del concepto de proporcionalidad, además de favorecer
el desarrollo de la autoestima del alumno, porque al resolver los problemas se
gana el reconocimiento y respeto de sus compañeros.

78
Bibliografía

Argudín, Y. (2009). Educación basada en competencias: Nociones y


antecedentes. México: Trillas.

Ahumada, P. (2005). Hacia una evaluación auténtica del aprendizaje. México:


Paidós.

Barriga, Á. D. (2006). El enfoque de competencias en la educación. ¿Una


alternativa o un disfraz de cambio? Perfiles Educativos , 7-36.

Brousseau, G. (2007). Iniciación al estudio de la teoría de las situaciones


didácticas. (D. Fregona, Trad.) Buenos Aires: Libros del Zorzal.

Delors, J. (1996). Los cuatro pilares de la educación. En J. Delors, La educación


encierra un tesoro (págs. 89-103). México: UNESCO.

Gálvez, G. (1994). La didáctica de las matemáticas. En C. Parra, & I. E. Saiz,


Didáctica de las matemáticas: aportes y reflexiones (pág. 299). México: Paidós
Educador.

Guevara Mora, G. (2010). Aprendizaje basado en problemas como técnica


didáctica para la enseñanza del tema de la recursividad. InterSedes: Revista de
las Sedes Regionales, 142 - 167

Maldonado Pérez, M. (2007). El trabajo colaborativo en el aula universitaria.


Laurus, 13(23), 263 - 278

Mochón Cohen, S. (2012). Enseñanza del razonamiento proporcional y


alternativas para el manejo de la regla de tres . Educación Matemática , 133-157.

Puing, L. (1997). Análisis fenomenológico. En L. Puing, La educación matemática


en la enseñanza secundaria (págs. 61-94)

Sadovsky, P. (2005). la teoría de la situaciones didácticas: un marco para pensar y


actuar la enseñanza de la matemática. En Reflexiones teóricas para la Educación
Matemática. Buenos Aires: Libros del Zorzal.

Santos Trigo, L. M. (1997). Principios y métodos de la resolución de problemas en


el aprendizaje de las matemáticas. México: Iberoamérica.

SEP. (1995). La enseñanza de las matemáticas en la escuela primaria. Taller para


maestros, segunda parte. México: SEP.

79
SEP. (1994). Libro para el maestro. Matemáticas. Secundaria. México: SEP.

SEP. (2011). Plan de estudios. Educación Básica. México: Sep.

SEP. (2006). Programa de estudios. Matemáticas. México: SEP.

Zabala, A., & Arnau, L. (2007). 11 ideas clave: cómo aprender y enseñar
competencias. Barcelona: Graó.

Referencias electrónicas

OCDE. (2006). PISA 2006. Marco de la evaluación. Recuperado el 27 de 05 de


2014, de http://www.oecd.org/pisa/39732471.pdf

Roegiers, X., & Peyser, A. (25-28 de Junio de 2007). BIEF. Recuperado el 27 de


05 de 2014, de
http://www.ibe.unesco.org/fileadmin/user_upload/COPs/News_documents/2007/07
10SanJose/evaluacion_de_competencias.pdf

SEP. (2011). ENLACE 2006-2011. Recuperado el 27 de 05 de 2014, de Medias y


Niveles de logro por entidad federativa.:
http://www.enlace.sep.gob.mx/content/ba/pages/estadisticas_2011/estadisticas.ht
ml

UNESCO. (Septiembre de 1994). OEI. Recuperado el 27 de 05 de 2014, de


http://www.oei.es/quipu/marco_jomtien.pdf

UNESCO. (26-28 de 04 de 2000). Foro Mundial sobre la Educación. Recuperado


el 27 de 05 de 2014, de
http://unesdoc.unesco.org/images/0012/001211/121117s.pdf

80
ANEXO
ANEXO 1

Rúbrica para la evaluación de las situaciones didácticas

Requiere
Criterios Satisfactorio Suficiente Insuficiente
Apoyo
Reconoce lo Identifica el No identifica el
Identifica una
que el problema y Identifica el problema o lo
parte del
problema sus problema hace
problema
plantea características incorrectamente
Hace una
Identifica los Elaborar una Hace una lista
Hace una lista
datos del lista de todos incorrecta de
lista de datos incompleta
problema los datos datos
de datos
Determina
Conoce lo que Reconoce lo
cual es la Tiene la idea No sabe que es
el problema le que le
incógnita y de que va a lo que le solicita
solicita solicita el
como la va a hacer el problema
(Incógnita) problema
resolver
Genera Predice
Predice todos Predice las No logra realizar
posibles algunos
los posibles hipótesis una predicción
hipótesis factores
Elaborar una
Elabora una
lista con todos Elabora una Elabora una
lista con
Realiza el los pasos y lista con lista incorrecta
algunos
procedimiento toma en todos los de pasos
pasos
cuenta pasos
detalles
Presenta de Presenta los Presenta los
Llega al forma escrita y resultados de resultados de Sus resultados
resultado gráfica los forma forma son incorrectos
resultados organizada incompleta
Obtiene
conclusiones
correctas y Llega a Llega a
No logra realizar
Conclusiones crea nuevos conclusiones algunas
conclusiones
conocimientos correctas conclusiones
y nuevas
hipótesis

81
Anexo 2

Escala de rango

Aspectos a evaluar 1 2 3 4 5

Explica claramente el problema


Explica además de los pasos, sus ideas
Presenta más de una solución
Si reciben una respuesta incorrecta, la usa
para crear una discusión
Realizan buenas preguntas a la clase,
tales como: ¿será esta la única manera de
hacerlo?, ¿es esta la única respuesta
posible?, ¿qué pasa si...?
Responden las preguntas realizadas por
sus demás compañeros/as
Está atento a la clase y respeta la
participación de sus compañeros

1= Nunca

2= Raramente

3= Algunas veces

4= Casi siempre

5= Siempre

82
Anexo 3

Hoja de observación para el trabajo por equipos

Equipo Equipo Equipo Equipo Equipo


Criterios
1 2 3 4 5
Intercambian ideas antes de
hacer las pruebas
Colaboran en la elaboración de
las pruebas
Atienden y respetan las
opiniones de los demás
Utilizan los materiales con
precaución
Proponen explicaciones de lo
que observan
Aplican términos científicos en
sus explicaciones
Registran y sistematizan sus
observaciones

Acotaciones:

NS (No Suficiente)

S (Suficiente)

B (Bien)

MB (Muy bien)

83
Anexo 4

Registro anecdótico

FECHA: ____________ TAREA: _______________________________________

DOCENTE: ________________________________________________________

RECUPERACIÓN DE
AVANCES,
REGISTRO DE ACTIVIDADES
DIFICULTADES Y
APOYOS REQUERIDOS

84
DIAGNÓSTICO

1. ¿Cuál es la fórmula para calcular el área de un triángulo?

𝑏 ℎ
𝑎 2
𝑏 𝑏 𝑐 𝑏 𝑎 𝑑 𝑎𝑏
En este reactivo lo que espero es conocer si los alumnos reconocen cuál de las
formulas anteriores me permiten saber la superficie de un triángulo.

a) Es la respuesta correcta
b) El alumno tiene una noción acerca de la obtención de la superficie del
triángulo, pero no recordó que se tiene que dividir entre dos, debido a que
obtenemos un rectángulo con dos triángulos o la mitad del rectángulo y la
formula de este es
c) Esta respuesta tiene la misma justificación que la anterior sólo que en lugar
de colocar la letra que denota a la altura (h) coloca la letra a (que
significaría altura)
d) En este reactivo es erróneo por exceso y reducción a la vez, al contener al
área de más y hacer falta la división entre dos

2. Un objeto que en la Tierra pesa 4 kilogramos, en Júpiter pesa 10 kilogramos, ¿cuánto


pesará en Júpiter un objeto que en la Tierra pesa 12 kilogramos?
a) 6 kilogramos b) 18 kilogramos c) 30 kilogramos d) 4.8 kilogramos

En este reactivo, lo que espero es conocer el grado de dominio acerca de la


proporción, que se ocupa en gran medida en la Física, si los alumnos dominan la
regla de tres, o en su caso el valor unitario.

a) La respuesta es incorrecta, el alumno sólo se percató que existe una


diferencia de 6 kg entre la tierra y júpiter, pensando así que a Júpiter le
resta 6 para obtener la respuesta, pero en este caso tampoco existió una
buena lectura, porque los 12 kilogramos son en la Tierra no en Júpiter.
b) La respuesta es incorrecta, en este inciso el alumno se pudo percatar de
que existe una diferencia de 6, pues si a cuatro le suma 6 obtiene como
resultado 10, así que, sí a 12 le suma 6 obtiene 18 como resultado, sin
embargo debe existir cierta proporción.
c) La respuesta es correcta
d) En esta respuesta, el alumno realizo una regla de tres, en la cual le fallo en
las operaciones pues en lugar de multiplicar 10 x 12, el multiplico 12 x 4 y
posteriormente lo dividió entre 10, y no debió ser así, considero que ubico
mal los datos.

85
𝑥+6
3. Cuál es el valor de x, en la siguiente expresión 2
5
a) 5 b) 4 c) 8 d) 1

En este reactivo, podre percatarme acerca de cómo el alumno resuelve una


“formula” cuando no conoce el despeje, pues el resultado ya está dado para esa
fórmula, sin embargo existe una variable que no está definida.
a) Algunos alumnos, no leen correctamente por lo que al solicitarles el
resultado, ellos observan que ya esta dado y colocan ése como el correcto.
b) Este es el resultado correcto
c) En esta opción el estudiante puede, en su concepción realizar la jerarquía
de operaciones por lo cual si 8 lo divido en dos, son cuatro le suma seis y
posteriormente lo divide entre dos, da como resultado cinco.
d) En este punto el alumno no tiene bien entendido lo que es el inverso
multiplicativo, al no enviar al 2 otro extremo multiplicando, sino sumando
por lo que 5 +2 son 7 y un número que sumando 6 me de 7 pues es uno
4. Para obtener el volumen de un pirámide cuadrangular que el lado de su base mide 5 cm
y la altura del pirámide es de 10 cm, ¿cuál de las siguientes opciones corresponde al
planteamiento?
5 5 10
a) 𝑣
3
b) 𝑣 5 5
5 2 10
c) 𝑣
3
d) 𝑣 5 5

Lo que espero en este reactivo es saber si el alumno sabe sustituir correctamente


una fórmula a partir de los datos dados en el problema.

a) La respuesta es correcta
b) Conoce la fórmula, sin embargo olvido que al obtener el volumen este se
tiene que dividir entre tres porque es una pirámide.
c) Aquí la forma tiene coherencia, lo que omitió es que el área de un cuadrado
se saca multiplicando lado por lado o elevando al cuadrado la medida del
lado de la base, y en este punto multiplica por dos en lugar de multiplicar
dos veces lado por lado.
d) En este punto en lugar de dividir multiplica por tres, sabe que tiene que
realizar una operación con el tres, pero olvida que es la división, debido a
que la pirámide es la tercera parte del volumen de un prima con la misma
base y altura

86
5. Dos decímetros son equivalentes a:
a) 2 cm b) 200 cm c) 0.2 cm d) 20 cm

Conocer si el alumno tiene una noción acerca de múltiplos y submúltiplos.

a) Es incorrecto debido a que nuestra unidad de medida es el metro y un


metro tiene 100 cm, y un decímetro equivale a 10 cm
b) Es incorrecto, pues 200 cm es equivalente a dos metros y solicitan
decímetros
c) Esta respuesta nos indica un submúltiplo inferior es decir 2 mm
d) Respuesta correcta.

6. Daniela y Octavio decidieron realizar un recorrido en bicicleta, en una pista de 400


metros para ver quién llegaba primero a la meta. Sí Daniela avanza 500 metros en un
minuto y Octavio 400 metros en un minuto, Daniela decide dar una ventaja de 300
metros a Octavio. ¿Quién ganará la carrera?

Este reactivo, considero que es muy rico, debido a que el alumno con todas las
herramientas matemáticas que posee puede dar solución, me agrado porque
espero que el alumno indague la respuesta, a través de sus estrategias, sirviendo
así en Física para los despejes de las formulas, el alumno a partir de la fórmula
busca una forma de dar solución al problema. En este reactivo puede utilizar una
recta numérica, sumas, multiplicaciones o divisiones, además de una función lineal

7. Un automóvil recorre 40 km con 8 litros de gasolina, cuántos km recorrerá con 10, 16 y


22 litros de gasolina. Grafica tu respuesta

Al igual que en algunos reactivos anteriores, se trata de proporcionalidad, sólo que


en este caso tendrán que encontrar la constante de proporcionalidad o valor
unitario, posteriormente observaré si su grafica fue la correcta o no. Además de
adentrarlo con la noción de velocidad

8. Traza una recta de 5 centímetros, coloca a cada uno de los extremos A y B según
corresponda, después ubícate en A y con tu transportador traza a 60° una recta de 3 cm,
coloca C.

Este reactivo es abierto, pues me permitirá conocer si el alumno sabe utilizar


correctamente el transportador, y traza correctamente lo que se solicita. De tal
forma que mediré a cada uno de los trazos y ángulos marcados

87

También podría gustarte